0% found this document useful (0 votes)
8 views

Motion Book Notes

The document discusses the concepts of linear motion, including distance, displacement, speed, and velocity, highlighting their definitions and differences. It explains kinematics and dynamics, providing examples of linear and non-linear motion, and outlines how to calculate average speed and velocity. Additionally, the document covers acceleration, deceleration, and the use of ticker-timers to measure motion variables.

Uploaded by

shreyram25
Copyright
© © All Rights Reserved
Available Formats
Download as PDF, TXT or read online on Scribd
0% found this document useful (0 votes)
8 views

Motion Book Notes

The document discusses the concepts of linear motion, including distance, displacement, speed, and velocity, highlighting their definitions and differences. It explains kinematics and dynamics, providing examples of linear and non-linear motion, and outlines how to calculate average speed and velocity. Additionally, the document covers acceleration, deceleration, and the use of ticker-timers to measure motion variables.

Uploaded by

shreyram25
Copyright
© © All Rights Reserved
Available Formats
Download as PDF, TXT or read online on Scribd
You are on page 1/ 31

2.

1 Analysing Linear Motion


2 Both distance and displacement have the same SI units.
2.1 Analysing Linear Motion They are measured in metres (m).
3 Both speed and velocity have the same SI units of metre per An object moves from position
Linear Motion second (m s–1). A to B. The figure below
4 However, since displacement and velocity are vector quantities, the shows five possible paths
direction of motion must be stated together with the magnitude. taken by the object. Which is
1 Linear motion is motion in a straight line. 3 Examples of non-linear motion: the shortest path?
2 Examples of linear motion: Earth Q

R B

Sun 1
P X
A
Figure 2.3 shows the location of two towns, P and Q.
F Y F
N
O (a) A spinning top (b) The earth orbiting lake O
2

2
R the sun The path with the shortest R
CHAPTER

CHAPTER
M
(a) A passenger on a moving escalator Figure 2.2 Jamil's length is APB (the straight line M
Town Q
car that joins A and B) and APB is
4 Raven's 4
4 The study of the motion of an object without helicopter known as the displacement
considering the forces acting on it is called Town P from A to B.
kinematics. The magnitude of the
mountain displacement is the shortest
5 The study of the motion of an object and
Figure 2.3 distance that links the initial
(b) An athlete running a 100 m race the forces acting on the object is called
and final positions of an
Figure 2.1 dynamics. Jamil drives a car along the road from town P to town Q, which is 300 km away. object that has moved.
The journey takes 5 hours. Raven flies a helicopter due east from town P to Q The other paths, AQB, ARB,
for a distance of l00 km in half an hour. AXB and AYB are known as
Distance, Displacement, Speed and Velocity Analyse, in terms of speed and velocity, Jamil's and Raven's journeys. the distances travelled from A
to B.
1 The physical quantities of motion are distance, displacement, speed, velocity, time and Solution
acceleration. Jamil’s journey Raven’s journey
Motion Distance travelled = 300 km Displacement = 100 km due east
An object in motion changes position. Time taken = 5 hours Time taken = 0.5 hour
Distance travelled Distance in a
Speed, v = ________________ Displacement
Velocity, v = ____________ specific direction.
Time taken Time taken
Distance Displacement 300 km
_______ 100 km
• The total length of the path an object travels from • The distance an object travels in a specific
= = _______
5h 0.5 h
one location to another. direction.
= 60 km h–1 = 200 km h–1
• Distance is a scalar quantity. • The magnitude of displacement is equal to the
shortest distance between two points. The speed of the car is 60 km h–1. The velocity of the helicopter is 200 km h–1 due east.
• Thus, displacement is a vector quantity.

Note: SI unit is not used in this example.


Speed and velocity both describe how fast an object is moving but
there is an important difference between these two quantities, Average Speed and Average Velocity
i.e, velocity is related to direction. 2
1 Refer to Example 2.
An athlete runs 100 m in 10 s.
2 The term average velocity is used because the
Velocity What is his average velocity?
athlete is not running at a constant velocity of
Speed • Velocity is the speed of an object in a specified
• Speed is the distance covered per unit time, that is, direction, that is, the rate of change in 10 m s–1 in the whole race. Solution
the rate of change in distance. displacement. 3 Similarly, the speed of the car (in Example 1) is s
Total distance travelled, s (m) Distance moved in a specific direction an average speed since the speed of the car is Average velocity, v = __
Speed, v = _________________________ Velocity, v = ______________________________________ t
Time taken, t (s) Time taken not the same throughout the whole journey.
100 m
• Speed is a scalar quantity. Displacement, s (m)
= ____________________
4 Table 2.1 shows the difference between average = ______
Time taken, t (s) speed/average velocity and constant speed/ 10 s
• Velocity is a vector quantity. constant velocity. = 10 m s–1

Forces and Motion 38 39 Forces and Motion


Table 2.1
4
Average speed / Average velocity Constant speed / Constant velocity
A car moves at an average speed/velocity of A car moves at a constant or uniform speed / velocity Figure 2.4 shows a car moving round a roundabout. Solution
20 m s–1 (equivalent to 72 km h–1). of 10 m s–1. VR
VP (a) The speed of the car is 3 m s–1.
This means the car may move 15 m in the first This means the car moves 10 m in the first second, P (b) (i) At P:
R
second, 25 m in the next second and 20 m in the 10 m in the next second, 10 m in the third second, The velocity, vP = 3 m s–1 due east
third second. and so on. (ii) At Q:
On average, the car moves a distance/displacement The car always covers a distance or displacement The velocity, vQ = 3 m s–1 due west
of 20 m in 1 second for the whole journey. of 10 m in 1 second for the whole journey. N Q (iii) At R:
The magnitude of speed/velocity remains the same. VQ The velocity, vR = 3 m s–1 due N 45° W
15 m 25 m 20 m
F Figure 2.4 Note: F
t=0 t=1s t=2s t=3s 10 m 10 m 10 m
O If the car covers a distance of 3 m each second, find The speed remains constant throughout while the O
2

2
R t=0 t=1s t=2s t=3s (a) the speed of the car, velocity is changing (i.e. change in direction, but not R
CHAPTER

CHAPTER
M (b) the velocity of the car at in magnitude, bearing in mind that velocity is a vector M

4
(i) P (ii) Q (iii) R quantity which has both magnitude and direction.) 4

Acceleration and Deceleration


An object is said to be moving at a constant or uniform speed if it moves
equal distances in equal successive time intervals, no matter how small the 1 When the velocity of an object changes with 3 Since velocity is a vector quantity (i.e., it has
time interval is. time, the object is said to be accelerating. magnitude as well as direction), acceleration
For example, an object moving at a constant speed of 10 m s–1 covers a 2 Acceleration is defined as the rate of change is, thus, a vector quantity.
distance of 10 m every second, or 1 m every 0.1 s. in velocity with time. 4 The SI unit for acceleration is metre per
Acceleration, a Initial velocity = u second per second or m s–2 (read as metre per
Change in velocity Final velocity = v second squared).
= Time taken = t
Time taken
SPM
3 ’07/P1 Final velocity – Initial velocity
=
Time taken
In an activity for a Physics lesson, a student was (a) Total distance travelled = AB + BC
instructed by his teacher to run due north for = 12 + 16 ∴ a = v–u
a distance of 12 m before moving east for another = 28 m t
16 m. The time taken was 20 s. Total distance travelled v–u
Average speed, v = _____________________ v>u Acceleration, a = v<u
What was the student’s Time taken t
(a) average speed, and 28 m
(b) average velocity? = _____ Acceleration Deceleration/Retardation
20 s
= 1.4 m s–1 • When v > u, a is positive, the velocity is increasing. • When v < u, a is negative, the velocity is decreasing.
Solution (b) Displacement • Thus, the speed of the object increases and is said • Thus, the speed of the object slows down and is
= The distance travelled in the direction of AC to be accelerating. said to be decelerating.
16 m For example, For example,
B C = 122 + 162 Apply Pythagoras’ theorem u=0 v = 30 m s–1 u = 30 m s–1 v=0
= 20 m 20
25
30 35
40 20
25
30 35
40 20
25
30 35
40 20
25
30 35
40

16
45 45 45 45
To determine the
tan θ = __
15 15 15 15
50 50 50 50
10 10 10 10
direction of AC 55 55 55 55
12
5 m s–1 5 m s–1 5 m s–1 5 m s–1
0 60 0 60 0 60 0 60

= 1.333
12 m
tan θ = 53.1°
N Average velocity, v 50
55
24

1
2
60
27

18
0
3

5
1
3

12
6

9
5
10 50
55
24

21
60
27

18
30

15
3

12
6

9
5
10 50
55
4
2

21
60
27

18
30

5
1
3

12
6

9
5
10 50
55
4
2

21
60
27

18
30

5
1
3

12
6

9
5
10

Displacement
45 15 45 15 45 15 45 15

40 20 40 20 40 20 40 20
35 25 35 25 35 25 35 25
30 30 30 30

= t=0 t=5s t=0 t=5s


θ Time taken
The driver steps on the accelerator when the traffic The driver applies the brakes when he sees the
20 m
A = light turns green. The car increases its speed with traffic lights turn red to reduce its velocity with a
20 s
an acceleration of 6 m s–2. deceleration or retardation of 6 m s–2 until it stops.
= 1 m s–1 in the direction N 53.1° E Mathematically, we write as: a = 6 m s–2 Mathematically, we write as: a = –6 m s–2

Forces and Motion 40 41 Forces and Motion


2 The ticker-timer can be used to determine the (c) The velocity of the object
5 following variables. (d) The acceleration of the object
From a constant velocity of 2000 m s–1, the velocity of Acceleration, a (a) The time interval of the motion (e) The type of motion of the object
a rocket increases to 3000 m s–1 in 5 s when the rate of v–u (b) The displacement of the object
=
combustion in the combustion chamber is increased. t
What is the acceleration of the rocket? 3000 m s–1 – 2000 m s –1
= 2 1
5s A vibrating metal strip with a pin is set to vibrate A ticker-timer consists of an electrical
Solution
1000 m s –1 up and down 50 times per second (i.e., at 50 hertz, vibrator, which is connected to an alternating
= 200 metres per second per
5s second, i.e., a gain in velocity which is the frequency of the a.c. supply). current (a.c.) power supply (12 or 6 V).
200 m s –1 of 200 m s–1 in each second.
= 1 vibrator
1s 2 metal strip
u = 2000 m s–1 v = 3000 m s–1
F t=0 t=5s = 200 m s –2 Also read as 200 metres per F
O second squared. 50 dots are O
2

2
R punched on the R
tape in one second
CHAPTER

CHAPTER
M M
3
4 ticker-tape 4
inclined runway
The acceleration of the rocket in Example 5 is 200 m s–2. This means that its velocity increases by 200 m s–1 4 trolley
for every second, as illustrated in the following diagram. Figure 2.6

3 4
Each time the metal pin moves down, The ticker tape is attached to a trolley which moves on a
t=0 t=1s t=2s t=3s t=4s t=5s
v = 2000 m s–1 v = 2200 m s–1 v = 2400 m s–1 v = 2600 m s–1 v = 2800 m s–1 v = 3000 m s–1 it makes a dot on the carbonised ticker bench or runway. As the trolley moves, it pulls the tape
tape which passes underneath it. through the ticker-timer. A trail of dots is punched on
the ticker tape at equal time intervals. Thus, the dots on
the tape form a complete record of the motion of
6
the trolley.
Azmi cycles at a uniform speed of 20 m s–1. He then Solution
stops pedalling and his bicycle comes to a stop after Analysing Motion on a Single Strip
u = 20 m s–1, v = 0 m s–1 and t = 8 s
8 s. What is his average deceleration? v–u
u = 20 m s–1 v=0
Acceleration, a = ____ The type of motion of the trolley can be inferred from the distance between the dots on the ticker
t tape pulled by the trolley.
0 – 20
= _____ direction of motion
8 Negative means
= –2.5 m s–2 deceleration.
t=8s
stop pedalling bicycle stops Deceleration = 2.5 m s–2 The distance between two neighbouring dots is equal. ➟ The object is moving at a constant/uniform velocity.
Figure 2.5
direction of motion direction of motion

The distance between two neighbouring The distance between two neighbouring dots is
dots is small. greater.
change in displacement change in velocity ➟ The object is moving slowly. ➟ The object is moving at a greater velocity.
Displacement Velocity Acceleration
causes causes
direction of motion direction of motion

Study of Linear Motion SPM


’03/P3/(B)

The distance between two neighbouring The distance between two neighbouring dots
Ticker-timer dots increases. decreases.
➟ The velocity increases. ➟ The velocity decreases.
1 A ticker-timer is a device used in the laboratory to study the motion of a moving object, ➟ The object is decelerating.
➟ The object is accelerating.
usually a trolley.

Forces and Motion 42 43 Forces and Motion


To Find the Time Interval of Motion and Velocity of an Object SPM Time taken (from the midpoint of AB to the Change in velocity
’05/P3/(B) Acceleration, a = ________________
midpoint of PQ) to produce the change in velocity Time taken
1 The time for 1 dot-space, or 1 tick of time is the time interval between one carbon dot and = 5 × 0.02 s v–u
the next one on the ticker tape. = 0.1 s = ____
t
dot Watch out! It is 5-tick, not 6-tick. The average
(75 – 25) cm s–1
number 0 1 2 3 4 5 67 8 910 20 30 40 50 velocity happens somewhere at the midpoints = ______________
of AB and PQ respectively. Therefore, the time 0.1 s
direction taken for the change in velocity is from the
of one tentick midpoint of AB to the midpoint of PQ. 50 cm s–1
________
motion of time =
0.1 s
First 10-tick Second 10-tick Third 10-tick Fourth 10-tick Fifth 10-tick
strip strip strip strip strip
= 500 cm s–2
= 5 m s–2
Figure 2.7
F
Alternative method F
O Since the vibrating pin makes 50 dots in 1 4 Therefore, one 10-tick of time O
2

2
R second, therefore: = 10 × 0.02 s = 0.2 s The time taken, t to produce the change in velocity can also be found as shown below. R
∴ 10-tick = 0.2 s
CHAPTER

CHAPTER
M Time for moving 50 dot-spaces = 50 ticks = 1 s First, label the time as an interval of 0.02 s at the dots as shown in the figure below. M
1 5 Similarly, one 5-tick of time and one 2-tick of
4 ∴ 1 tick = s = 0.02 s 4
50 time can be calculated as shown in Table 2.2.
2 A 10-tick of time is the time interval from dot 6 With the quantities of time interval and direction
number 0 to dot number 10 on the tape. displacement, we can calculate the velocity of of motion
an object. 0 0.02 s 0.04 s 0.06 s 0.08 s 0.10 s 0.12 s
3 The next 10-tick of time is the time taken to
move from dot number 10 to dot number 20. t
0.01 s 0.11 s
Table 2.2

10-tick strip Time taken to move from A to B Constant velocity, ∴ t = 0.11 s – 0.01 s = 0.1 s
= 10 × 0.02 s s 8 cm
A direction of motion B v = =
10 = 0.2 s t 0.2 s
1 2 3 4 5 6 8 9
7
∴ 10-tick = 0.2 s = 40 cm s–1 Making a Tape Chart 2 On the tape, lines are drawn across dots
8 cm number 0, 10, 20 and so on, from the start of
5-tick strip Time taken to move from P to Q Average velocity, 1 Figure 2.9 shows a ticker tape obtained in an the first clear dot, to mark off in sections 10
direction of motion = 5 × 0.02 s s 6 cm experiment. dot-spaces long.
P Q v = =
1 2 3 4 5 = 0.1 s t 0.1 s dot 3 The 10 dot-spaces are labelled in order. The tape
number 01 2 3 4 5 6 7 8 9 10
6 cm
∴ 5-tick = 0.1 s = 60 cm s–1 20 30
is then cut at the lines to form 10-tick strips.
1 2 3
direction 4 The 10-tick strips are pasted in order side by
of motion one tentick
2-tick strip Time taken to move from R to S Average velocity, of time side on paper, preferably a graph paper, for
= 2 × 0.02 s s 5 cm easy measurement, to form a tape chart as
R direction of motion S v = =
= 0.04 s t 0.04 s Figure 2.9
1 2 shown in Figure 2.10.
∴ 2-tick = 0.04 s = 125 cm s–1 v
5 cm Velocity, v (or distance moved per tentick)

To Find the Uniform Acceleration or Deceleration of an Object 8


7
6

7 4
5

3
2
A trolley is moving down a runway. A strip with six The ticker-timer vibrates at a frequency of 50 Hz. 1
dot-spaces as shown in Figure 2.8 is obtained. Find the acceleration of the trolley.
Solution t
1 2 3 4 5 0 10 20 30 40 50 60 70 80 Time in ticks
direction
Frequency, f = 50 Hz 0 0.2 0.4 0.6 0.8 1.0 1.2 1.4 1.6 Time in seconds
of motion A B P Q ∴ 1 tick = 0.02 s Figure 2.10
0.5 cm
Average velocity from A to B: u = = 25 cm s–1
0.02 s 5 The length of each 10-tick strip is the distance the time taken for each strip is the same, i.e.
1.5 cm
0.5 cm
1.5 cm moved in one 10-tick of time, that is, 0.2 s. 0.2 s.
Average velocity from P to Q: v = = 75 cm s–1 6 The length of the strip represents velocity. The 7 As such, the tape chart is practically a velocity-
Figure 2.8 0.02 s
longer the strip, the greater the velocity since time graph where the vertical axis is the

Forces and Motion 44 45 Forces and Motion


velocity (distance moved per 10-tick), while length actually represents the velocity since the
the horizontal axis indicates the time since time taken for each strip is the same.
each strip starts 0.2 s after the one before. 9 Tape charts can also be made of strips with
To prepare a friction-compensated runway
8 Some may prefer to label the vertical axis as 5 dot-spaces or 2 dot-spaces. In Activity 2.1, there are two main forces acting on the trolley along the runway, i.e., the component weight of the trolley
length of the strips, but remember that this down the runway and the frictional force of the runway on the trolley (please refer to Section 2.9 on page 118).
(a) If the slope is not steep enough, (b) If the slope is too steep, the trolley (c) The slope is adjusted until the trolley
after a slight push, the trolley moves moves down the runway by itself. goes down at uniform velocity, after
a short distance and then stops. The dots get farther apart. a slight push. The dots on the tape
Activ To determine displacement, velocity and acceleration of a trolley
ity 2.1 The dots on the tape get closer. are equally-spaced. This is a
direction of motion friction-compensated runway.
Apparatus/Materials Calculation direction of motion
direction of motion
trolley given a slight push
Trolley, runway, ticker-timer, 12 V power supply, (a) Displacement = x1 + x2 + x3 + x4 + x5 + x6
F ticker tape, cellophane tape and wooden block. stop trolley goes down on its own F
O where x is the length of the strip. velo
trolley given a slight push
O
(b) Time taken = 6 × 10-tick
2

2
Arrangement of apparatus city c on s
R Key : incr tant ve R
e as locity
= 6 × 0.2 s ing
CHAPTER

CHAPTER
M component weight friction M
ticker tape ticker-timer
= 1.2 s
4 trolley 4
Displacement
inclined Average velocity = ____________
12 V a.c.
power runway Time taken 8
supply wooden
block (x + x + x + x + x + x ) cm
_________________________ A student carried out an experiment using a trolley Average velocity
= 1 2 3 4 5 6
Figure 2.11 1.2 s and a ticker-timer that vibrates at a frequency of Total distance travelled
= ____________________
(c) Average velocity for the 1st strip: 50 Hz. Figure 2.13 shows a tape chart consisting of Time taken
Procedure
x1
___ 10-tick strips that he obtained. 30 cm
1 The apparatus is set up as shown in Figure 2.11. u= cm s–1 = ______ = 30 cm s–1
0.2 velocity (distance moved per tentick)
1.0 s
2 The inclination of the runway is set so that the
trolley will roll down freely after it is released. Average velocity for the 6th strip: 4 = 0.30 m s–2
3 A length of ticker tape is passed through the x6 (c) Average velocity for the 1st strip:
v = ___
10
cm s–1 3 2 cm
ticker-timer and attached to the trolley. 0.2 u = ____ = 10 cm s–1
8 0.2 s
4 The ticker-timer is switched on and the trolley is 2
released. Time taken for the change in velocity, t 6
Average velocity for the 5th strip:
5 The ticker tape obtained is then cut into 6 pieces 1 10 cm
= (6 – 1) × 0.2 s v = ______ = 50 cm s–1 Watch out, not 5 × 10-tick!
of 10-tick strips. = 5 × 0.2 s 4 0.2 s The time taken for the
6 The strips are pasted side by side on a graph = 1.0 s Time taken, t change in velocity is from
2
paper to form a tape chart. = 4 × 10-tick the midpoint of the 1st strip
to the midpoint of the 5th
Results Acceleration of the trolley is calculated from the 0 = 4 × 0.2 s = 0.8 s strip.
0 10 20 30 40 50 Time in ticks
velocity (distance moved per tentick) formula: 0 0.2 0.4 0.6 0.8 1.0 Time in seconds OR:
v–u t Time taken, t
5 a= 0.1 0.9 From the time axis.
t = (0.9 – 0.1) s = 0.8 s
x6
6 Figure 2.13 Change in velocity
∴ Acceleration, a = _________________
4
x5 Discussion Find
Time taken
3
5
(a) the total distance travelled,
The length of strip increases uniformly. v____
–u
x4 (b) the average velocity, =
2
4
Thus, the trolley moves down the runway at a (c) the acceleration of the trolley. t
x3 constant or uniform acceleration. (50 – 10) cm s–1
1
3
Solution = ______________
0.8 s
x2
2 Conclusion (a) The total distance travelled = 50 cm s–2 = 0.5 m s–2
The average velocity and acceleration of a trolley are (from dot number 0 to number 50)
x1 Note: Since the motion is of uniform acceleration, the
1
thus determined. = (2 + 4 + 6 + 8 + 10) cm
average velocity in (b) can also be calculated
Activity 2.1

= 30 cm u+v
0 10 20 30 40 50 60 Time in ticks
(b) Total time taken from the formula –v = .
0 0.2 0.4 0.6 0.8 1.0 1.2 Time in seconds The total time taken 2
t = 5 × 10-tick is 1 s if you label the
= 5 × 0.2 s (10 + 50) cm s–1
Figure 2.12
time axis in seconds. v– = = 30 cm s–1
= 1.0 s 2

Forces and Motion 46 47 Forces and Motion


The Equations of Linear Motion
In the questions, the vertical axis may not be labelled as As explained earlier, the distance between dots, or the For linear motion with uniform acceleration, a, the displacement or Summary of the equations for
velocity or distance moved per 10-tick (or 5-tick or
length of a strip, represents the velocity of a moving distance travelled, s, in a particular direction is given by: linear motion with uniform
2-tick). However, the way to solve the problems is still
object. Therefore, the increase in distance between dots s = Average velocity × Time taken acceleration:
the same.
and the increase in length between successive strips v = u + at
represent the increase in velocity. If the increment is
∴ s = 1 (u + v)t 1
s = (u + v) t
uniform, then the increase in velocity is uniform, that is, 2 2
the object is moving with uniform acceleration. 1 2
(a) In the figure below, the increase in distance s = ut + at
2
9 between the successive dots is equal (i.e., 0.2 cm). rearrange v–u rearrange v–u
v = u + at a= t = v 2 = u2 + 2as
Therefore, the acceleration is uniform. t a
Figure 2.14 shows a tape chart for a trolley which
F substitute substitute s = displacement F
O
moves up on an inclined plane. Determine the direction of motion u = initial velocity O
2

2
R deceleration of the trolley. v = final velocity R
1.0 cm 1.2 cm 1.4 cm 1.6 cm 1.8 cm 2.0 cm 1 1 a = uniform acceleration
( )
CHAPTER

CHAPTER
M Length of strips (cm) s= (u + v)t 1 s = (u + v) t M
2 s = u+v t 2 t = time interval
1
(b) In the figure below, the increase in length between 1 2
4 2 = (u + u + at)t 1 (v – u) 4
6 the successive strips is equal (i.e., 1 cm). Therefore, 2 = (u + v)
3
2 a
5 4 the acceleration is uniform. (Alternatively, if a 1
= (2ut + at 2) (v 2 – u2)
straight line can be drawn across all the top = 1
4 5
2
3 midpoints of the strips, the acceleration is uniform) 2 a
1 2
2
Velocity (cm per tentick) s = ut + at 2as = v 2 – u2
1
Velocity 2
8
7 v 2 = u 2 + 2as
0 0.04 0.08 0.12 0.16 0.20 0.24 Time (s) 6
t 5
0.02 0.22 4
3
Figure 2.14 2 There are 5 physical quantities in the equations of
1 1
Time (s) linear motion with uniform acceleration. (II) For s = (u + v) t:
Solution Time 2
0 0.2 0.4 0.6 0.8 1.0 1.2 Each of the four equations of motion involves 4 physical
The time of each 2-tick strip = 2 × 0.02 s quantities. The velocity of a rocket travelling at 2000 m s–1
= 0.04 s (c) In the figure below, there is no increase in length To solve numerical problems, you need to know 3 increases to 6000 m s–1 after moves through a
6 cm between the successive strips. Therefore, the quantities before you can find the value of the fourth distance of 80 km. Calculate the time for the rocket
u = = 150 cm s–1 acceleration is zero (the object is moving with to reach this velocity.
0.04 s quantity. In the beginning, you may be unsure of which
constant or uniform velocity). equation to choose to solve the problem. You may start 2000 m s–1 6000 m s–1
1 cm
v = = 25 cm s–1 by trial and error. After some practice, you will be able to
0.04 s Velocity (cm per tentick)
Velocity select the required equation easily.
80 km
Time taken to produce the change in velocity, t 6 The table below shows some examples.
= 5 × 0.04 s
= 5 × 2-tick (I) For v = u + at: Solution
= 0.20 s
Time(s) A car accelerates from 20 m s–1 with an acceleration u = 2000 m s–1, v = 6000 m s–1,
OR: t = (0.22 – 0.02) s Time
If the time axis is 0 0.2 0.4 0.6 0.8 1.0 of 2 m s–2. s = 80 km = 80 000 m, t = ?
= 0.20 s labelled. Unable to
What is the velocity after 8 seconds?
Acceleration, a ✓ ✓? ×? calculate t
(d) In the figure below, the decrease in length between 1 because only
v____
–u Solution First trial: s = ut + at2
= the successive strips is equal (i.e., 2 cm). Therefore, 2 2 quantities
t the deceleration is uniform. u = 20 m s–1, a = 2 m s–2, t = 8 s, v = ? are known.
(25 – 150) cm s–1 ✓ ✓ ✓?
= 0.2 s Velocity (cm per 5-tick) Velocity ? ✓ ✓× Unable to
1
calculate v, Second trial: s = (u + v) t 3 quantities
16 First trial: v 2 = u 2 + 2as because only 2 are known.
–125 cm s–1 14
Can be used
= 0.2 s 12 2 quantities are (2000 + 6000)
10 ? ✓ ✓✓ known. 80 000 = t to find t.
8
2
= – 625 cm s –2 Second trial: v = u + at
6 t = 20 s
= – 6.25 m s–2 4 v = 20 + 2(8) 3 quantities are
2 Time(s) = 36 m s–1 known. Can be
∴ Deceleration = 6.25 m s–2 0 Time used to find v.
0.1 0.2 0.3 0.4

Forces and Motion 48 49 Forces and Motion


10 12 2.1

Starting from rest, a sprinter reaches his top velocity By applying the brakes, a driver reduces the 1 During a class activity, Hashim walks 20 m due east. 6 When a dart is blown from a blowpipe of 1.2 m
in 3 seconds. He runs a distance of 24 m in the velocity of his car from 20 m s–1 to 10 m s–1 after He reverses his direction and walks 12 m. After that, length, it travels at a speed of 15 m s–1. Find the
he reverses his direction again and walk for another time taken for the dart to travel in the barrel.
3 seconds. What is his acceleration? a distance of 30 m. Calculate the deceleration of
10 m. If the total time taken is 30 s, what is his
(Assume his acceleration is uniform.) the car. speed and velocity?
7 Daniel drives his car at a constant velocity of
20 m s–1. He steps on his brakes to reduce the
Solution 2 A speedboat moves due north for 12 km before velocity of the car to 10 m s–1 after travelling 30 m.
u = 0, s = 24 m, t = 3 s, a = ? Solution turning east for 8 km. Later, the speedboat moves Calculate the deceleration and the further distance
1 u = 20 m s–1, v = 10 m s–1, s = 30 m, a = ? south for 6 km. What is the displacement of covered before the car stops.
Applying s = ut + at 2: the speedboat from its original position? Find its
2 Applying v 2 = u2 + 2as: 8 An aeroplane needs a velocity of 33 m s–1 for
average velocity, in m s–1, if the total time taken is
1 102 = 202 + 2a(30) 30 minutes. take-off.
F 24 = 0(3) + a(3)2 F
2 100 – 400
O a= 3 Find the acceleration or deceleration for the tape O
2

2
2(30) v = 33 m s–1
R R
a = 2 × 24
charts below. The ticker-timer in use vibrates at a
CHAPTER

CHAPTER
M = –5 m s–2 M
9 frequency of 50 Hz.
u=0
Deceleration = 5 m s–2
4 = 5.3 m s–2 (a) Velocity (cm per 10 ticks) 4

s
10

1 SPM
Clone If the plane accelerates at 3 m s–2, calculate the
’07
minimum length of the runway needed by the plane
11 The figure below shows the route taken by a van 2 to reach the speed for take-off.
Time
–1 from town P to town S. 9 A baseball pitcher holds a baseball in his hand for
Salina is driving at a velocity of 10 m s .
3 m before the baseball is thrown at a velocity of
Seeing a cow in front, she brakes to stop her car. Q 3 km R (b) Length of strips (cm) 39 m s–1.
If the deceleration of the car is 2 m s–2, what is 1 km
the distance the car covers before it comes to a 12
S 3.0 m
10
halt?
5 km 8
6
Solution 4 39 m s–1
2
P Time
u = 10 m s–1 v=0

What is the displacement of the van? 4 A sports car accelerates from rest and covers a
A 4.0 km C 7.5 km distance of 90 m in 6 s. What is its acceleration?
s=?
B 5.0 km D 9.0 km
5 A cyclist starts from rest and reaches a velocity of Find the acceleration of the baseball before it is
Comments 20 m s–1 in 8 s. Calculate his acceleration. released.
–1 –2
u = 10 m s , v = 0, a = –2 m s , s = ?
Displacement is a vector quantity. It is the shortest
Applying v2 = u2 + 2as:
distance that links the initial position of an object
0 = 102 + 2(–2)s
to its final position. In the above question, the
4s = 100
100 displacement is the distance PS. Join P to S, and use 2.2 Analysing Motion Graphs
s= Pythagoras’ theorem to calculate the displacement.
4
= 25 m Q 3 km R 2.2 Analysing Motion Graphs
1 km 3 km 1 km
S
Motion Graphs
4 km
PS = 3 2 + 4 2
= 5 km 1 Graphs can be useful in studying motion. They show the changes in
P
the motion of an object with time.
Don’t forget to insert the negative sign for deceleration 2 There are two main types of linear motion graphs:
when doing the calculation. Answer B (a) the displacement-time graph
(b) the velocity-time graph

Forces and Motion 50 51 Forces and Motion


Displacement-time Graphs Velocity-time Graphs SPM
’03/P1
SPM
’04/P1
SPM
’05/P1
SPM
’07/P1
SPM
’08/P1
SPM
’09/P1

1 A displacement-time graph (s-t graph) is a graph that shows how the 1 A velocity-time graph (v-t graph) is a graph that shows the variance in the velocity of an object against
displacement of an object varies with time. Relationship between the time.
position of an object, 2 A car starts from rest and accelerates for 20 seconds until it reaches a velocity of 30 m s–1. The driver
2 Figure 2.15 shows a student cycles at a constant velocity from position
displacement, s, and velocity, v: maintains this velocity for 20 seconds. The velocity of the car is then reduced until it stops at t = 60 seconds.
A to reach position B, which is 300 m away, in 200 seconds. He rests
s = (+) 3 The graph in Figure 2.16 shows how the velocity of the car changes against time.
for 100 seconds at position B and then cycles back to position A using
v = (+)
the same straight path. He reaches position A after another 200 v = 0 m s-1 v = 30 m s-1 v = 30 m s-1 v = 0 m s-1
(object on the
seconds. right of O and
N t=0 t = 20 s t = 40 s t = 60 s
zero velocity — at rest moving starts stops
displacement (m)
due east) v (m s-1)
positive velocity O
F — moving in a fixed direction
negative velocity F
O — moving in opposite direction s=0 v=0 constant velocity O
B B
2

2
300 and stops at To determine the acceleration of the car
R s = (–) s = (+) 30 R
the starting point
CHAPTER

CHAPTER
M v = (–) (object on the • From conclusion 1: M
acceleration deceleration Acceleration, a
(object right with (positive ∆y ∆y (negative
4 ∆y = Gradient 4
on the respect to O) gradient) I II III gradient)
I II III (0 – 30) m s–1
left of = (60 – 40) s
∆x ∆x
A ∆x A O and v = (–) 0 20 40 60
t (s) The negative
O
time (s) moving (object moving = –1.5 m s–2 sign indicates
100 200 300 400 500
due due west) ∴ The deceleration of deceleration.
Figure 2.15 Figure 2.16 the car is 1.5 m s–2.
west)
To determine the acceleration of the car
• Using the formula:
• In Section III of the graph: Change in velocity
Conclusion 3
Acceleration =
From conclusion 1: Time taken
• In Section I of the graph: Velocity = Gradient v–u A negative gradient indicates deceleration.
To find the velocity, use the formula: Acceleration, a =
(0 – 300) m t
Change in displacement =
Velocity = (500 – 300) s (30 – 0) m s–1
Time taken = 20 s • Distance travelled from t = 20 s to t = 40 s:
= – 1.5 m s–1 Distance = Velocity × Time
300 m = 1.5 m s–2
Velocity, v = The negative sign shows that the direction of = 30 m s–1 × 20 s
200 s ∆y
motion is opposite to its original direction. • Gradient of the graph = = 600 m
∆x • Area under the graph (for section II)
= 1.5 m s–1 Take note that velocity is a vector quantity.
(30 – 0) m s–1 = 30 × 20
Gradient of the graph • At t = 500 s, the graph intersects the t-axis. = = 600 unit2
(20 – 0) s
∆y The displacement at this moment is zero, that is,
= the student has returned to the original position. = 1.5 m s–2
∆x
(300 – 0) m
= Conclusion 4
(200 – 0) s Conclusion 1
= 1.5 m s –1 On a velocity-time graph, the area under
• In Section II of the graph, a horizontal line is drawn from On a velocity-time graph, the gradient of the the graph is numerically equal to the distance
t = 200 s to t = 300 s. During this period, the student graph represents the acceleration of the object. travelled.
remained at position B, which is 300 m away from position
A, the origin, with a velocity of zero.
The car travels at a constant velocity of 60 m s–1
from t = 20 s to t = 40 s. A horizontal line is
shown in the graph. This method of calculating the distance travelled
can be applied to any velocity-time graph,
Conclusion 1 Conclusion 2 whether the velocity is constant or not.
On a displacement-time On a displacement-time graph, For example, in Section I of the graph, the area
1
graph, the gradient of the a horizontal line (gradient = 0) Conclusion 2 of the shaded triangle (= × base × height)
2
graph is equal to the shows that an object is On a velocity-time graph, a horizontal line equals 300. So, the car travelled a distance of
velocity of the object. stationary, i.e., not in motion. (gradient = 0) represents a constant velocity. 300 metres in the first 20 second of its motion.

Forces and Motion 52 53 Forces and Motion


Non-uniform Acceleration
13
Figure 2.17 shows the velocity-time graph of a Solution acceleration constant velocity
1 Figure 2.19 shows an
motorcycle travelling along a straight road between (a) The line OA shows that the motorcycle accelerates athlete running a 100 m
two traffic lights. uniformly from 0 m s–1 to 12 m s–1. track and the corresponding
The line BC shows that the motorcycle decelerates velocity-time graph of his
uniformly before coming to a rest. motion.
velocity (m s–1)
(b) The motorcycle is moving at a constant velocity 2 The athlete increases his
velocity, v (m s-1)
for 10 s (line AB of the graph). constant velocity velocity (accelerates) until
(c) Distance between the two traffic lights the maximum velocity.
A B = Area under the graph smaller gradient gradient = 0 He maintains the
12 – smaller acceleration zero acceleration maximum velocity to
1
F = (10 + 25) × 12 = 210 m finish the race.
F
O 2 O
2

2
R 3 The acceleration of the R
(d) From 0 s to 5 s: gradient of tangent
athlete can be determined
CHAPTER

CHAPTER
M Δv = acceleration at this instant M
12 – 0
C Acceleration = = 2.4 m s–2 by drawing tangents at the
4 time (s) 5–0 Δt 4
O 5 10 15 20 25 respective points and
From 5 s to 15 s: O
time, t (s) determining the gradients
t1 t2
Figure 2.17 Acceleration = 0 because the
of the tangents.
object is moving Figure 2.19
From 15 s to 25 s: at constant
(a) Explain the motion of the motorcycle as Acceleration velocity
represented by the lines OA and BC on the 0 – 12
= = – 1.2 m s–2
graph. 25 – 15
(b) What is the time interval during which the Thus, the acceleration-time graph is as shown below.
motorcycle is moving at a constant velocity? A comparison between the displacement-time graph and the velocity-time graph:
(c) What is the distance between the two traffic acceleration (m s–2)
Displacement-time graph Velocity-time graph
lights? 2.4
displacement
(d) Sketch an acceleration-time graph to represent velocity
the motion of the motorcycle between the two time (s)
–1.2
traffic lights.
A
time time

The object moves at a Non-horizontal


Non-uniform Velocity constant velocity.
The object moves at a constant acceleration.
straight line

displacement, s (m) 1 Figures 2.18(a) and (b) show the Represents the velocity of
positions of a ball falling from rest Gradient Represents the acceleration of the object.
the object.
50
55
24
27
60
30
3

6
5
10
and its corresponding displacement-
time graph.
21

t=0
9
18 12
15

45 15

40
35
30
25
20

The object is stationary. Horizontal line The object moves at a constant velocity.
s1 2 The ball covers a longer distance in
60
gradient is steeper the second second as compared with The object returns to its Intersection on
The object stops.
55 5

⇒ greater velocity
30
27 3

50
24

10

t=1s the first second. The ball is moving


21 9

original position. the time-axis


18 12
15

45 15

40
35
30
25
20
gradient
s2
= velocity with non-uniform velocity.
(at t = 1 s)
3 To determine the ball’s instantaneous Positive ⇒ The object moves
velocity, for example, at t = 1 s or in a specific Sign of the Positive ⇒ Acceleration
Δs t = 2 s, a tangent must be drawn at the direction. gradient Negative ⇒ Deceleration
60
respective points on the graph, as Negative ⇒ The object moves (positive or
Δt
55 5

negative)
30
27 3

in the opposite
24 6

50 10

t=2s
21 9

18 12

shown in Figure 2.18(b).


15

45 15

time, t (s)
40 20
35 25
30

O 1 2
4 The gradient of the tangent is equal direction.
(a) (b) to the velocity. Area under the Numerically equals the distance travelled by the
No significance
5 However, the stone is falling with graph object
Figure 2.18
uniform acceleration.

Forces and Motion 54 55 Forces and Motion


14 2 SPM
Clone
’11

The velocity-time graph in Figure 2.20 shows a particle (a) 10 s (from t = 0 s to t = 10 s, the velocity is The diagram shows the velocity-time graph of a toy A 0m C 9m
starting from rest and travelling east. positive). car in motion. B 6m D 15 m
v ( m s – 1)
(b) 4 s (from t = 16 s to t = 20 s, the velocity is
negative). velocity(m s-1) Comments
B
20 (c) Distance travelled while moving towards the 6 Displacement = Area above the time-axis – Area below
east, S1 = Area of triangle ABC 4 the time-
10
1 axis
C P R = × 10 × 20 2
A t (s ) 2 0 time (s)
1 1
5 10 16 18 2 0 = (6)(2 + 3) – (2)(6)
= 100 m -2
2 2
– 10
F Q Distance travelled while moving towards the west, = 15 m – 6 m F
-4
O S2 = Area of triangle PQR = 9m O
2

2
-6
= 1 × 4 × 10
R R
Figure 2.20
CHAPTER

CHAPTER
M 2 M
What is the displacement of the toy car in 6 s? Answer: C
(a) How long does the particle travel towards the east? = 20 m
4 4
(b) How long does the particle travel towards the west?
(c) Find the average speed and the average velocity. Total distance
Average speed =
Time taken
Solution
100 + 20
To understand the above situation better, please refer =
20 2.2
to the figure below which demonstrates the positions
of the particle that vary with time. = 6 m s–1 s (m)
1 The figure 3 The figure below shows the velocity-time graph of a
100 m Final displacement shows the 20
motorcycle starting from rest and travelling north.
v=0 v = 20 m s–1 v=0 Average velocity = displacement-
A B C Time taken 15
time graph v (m s–1)
100 – 20 10
t=0s t=5s t = 10 s = of a moving
5 12
N
v=0 v = –10 v=0 20 particle.
t (s)
R Q P = 4 m s–1 to the east O 3 10 15
t = 20 s t = 18 s t = 16 s
15 20
(Please note that in the situation discussed, the (a) What is the velocity of the particle in the initial O t (s)
20 m 6 10 13
particle is always to the east of its original period of 3 seconds?
Note: The particle is at rest from t = 10 s to t = 16 s. position.) (b) How long is the particle stationary?
(c) At what point in time does the particle return to –10
its original position?
(d) Calculate (a) What is the deceleration from t = 10 s to t = 13 s?
(i) the average speed, and (b) What is the displacement of the motorcycle
(ii) the average velocity of the moving particle. during the first 13 s?
Area under velocity-time graph (c) For how long is the motorcycle travelling towards
v 2 The the south?
An object moving at a decreasing acceleration is v (m s–1)
velocity- (d) What is the final displacement of the motorcycle
moving at a decreasing velocity. time graph at t = 20 s?
shows the 12 (e) What is the average velocity of the motorcycle for
A
movement the whole journey?
t
B of a particle.
The velocity of an object with a decreasing acceleration 4 A sports car starting from rest, accelerates uniformly
is always increasing. However, the rate of increase is t (s)
to 30 m s–1 over a period of 20 s. The car maintains
Area A (above the t-axis) getting smaller. O 3 6 10 the velocity for 30 s. The velocity is then reduced
= Distance travelled by the moving object in its original uniformly to 20 m s–1 in 10 s and then brought to
direction (a) What is the total distance, in m, travelled by the
particle in 10 seconds? rest after another further 10 s.
Area B (below the t-axis) Draw a velocity-time graph to represent the journey
(b) For how long is the particle moving with constant
= Distance travelled by the moving object in the as described above. From the graph, find
velocity?
opposite direction (a) the acceleration of the car for the first 20 s,
(c) Calculate the ratio of acceleration : deceleration.
Total distance travelled = Area A + Area B (d) Calculate the average velocity of the particle. (b) the distance travelled, and
Final displacement = Area A – Area B (c) the average velocity over the time described.

Forces and Motion 56 57 Forces and Motion


2.3 Understanding Inertia

2.3 Understanding Inertia 4 When a stationary bus starts to move forward with an
acceleration, the passengers are thrown backwards.
Concept of Inertia SPM
’05/P1
SPM
’09/P1
Explanation
The passengers in the bus are originally in a stationary
1 All objects tend to continue with what they are doing. state. When the bus starts to move forward with an
2 Newton's first law of motion (also known as the Law of Inertia) acceleration, the inertia of the passengers keeps them in
states that: their position. Thus, the passengers are thrown backwards.

Every object continues in its state of rest or uniform speed in


a straight line unless acted upon by an external force.

F 3 The tendency of an object to maintain its state of rest or uniform F


O motion in a straight line is called inertia. O
2

2
R 4 Only an external force (or a non-zero net force) can cause a change to R
CHAPTER

CHAPTER
M M
the state of motion of an object (either at rest or moving at the same
speed in a straight line). When a moving bus stops suddenly, the passengers lurch
4 5 4
5 The external force is a non-zero net force, if more than one external forwards.
force act on the object.
Explanation
SPM
Situations Involving Inertia ’09/P1 The passengers are in a state of motion when the bus is
moving. When the bus stops suddenly, the inertia of the
When a boy is riding a bicycle that runs over a stone, he is passengers keeps them in motion. Thus, the passengers
1
thrown forward and off the bicycle. lurch forwards.
Explanation
The stone abruptly stops the motion of the bicycle, but the
inertia of the boy keeps him in the forward motion and
throws him forwards.

A cardboard is placed on the rim of a glass and a coin is


2
placed on top of the centre of the glass. When the
cardboard
cardboard is pulled away quickly, the coin resting on the
coin
cardboard drops straight into the glass.
6 A weight is suspended by a fine thread and rests at point P.
Explanation
When the weight is raised and then released, the thread
The inertia of the coin maintains its state of rest. When the fine
snaps when it passes point P.
thread
card is pulled away, the coin falls into the glass due to gravity. thread
snaps Explanation
Note: The fine thread is able to support the weight when it is
If the card is pulled away slowly, the frictional force stationary. When the weight is allowed to fall, the weight
between the coin and the card causes the coin to accelerate P
maintains its inertia of moving downwards when it passes
so that it moves together with the card. However, if the card point P. The inertia of the weight causes an additional force
is pulled quickly, the time is too short for the friction to weight to exert on the thread, thus causing it to snap.
cause any appreciable movement of the coin.
3 If thread Y is pulled slowly, thread X will snap. If thread Y is If a book is pulled out very quickly from the middle of a
7

BO
OK
thread X pulled suddenly, then thread Y will snap.

BO
pile of books, the books above it will drop instead of

OK
(can withstand 10.5 N)

BO
OK
Explanation moving along with it.
When thread Y is pulled slowly, the additional force to the Explanation
weight (10 N)
weight causes thread X to snap when the tension exceeds The inertia of the books above keeps them in their original
thread Y snaps
the breaking force. When thread Y is pulled abruptly, the position.
weight maintains its state at rest due to inertia. Thread Y Again, if the book is pulled out slowly, the books above it will
thus stretches and snaps. (If the weight does move, it will move together with the book as explained in situation 1 .
stretch and snap thread X instead.)

Forces and Motion 58 59 Forces and Motion


SPM SPM
8 The decorative item hanging from the rear-view mirror Relationship between Mass and Inertia ’04/P1 ’07/P1, P2
swings backwards when a stationary car starts to move or
accelerates. 1 A shopper in a supermarket observes that it is always easier to start
Explanation moving an empty trolley than a full trolley.
The inertia of the decorative item keeps it in its original
position while the car moves forwards.

9 The decorative item swings forwards when the moving car


comes to a sudden halt.
Explanation
F Supertankers: Large crude oil F
O
The inertia of the decorative item keeps it in motion when carriers O
2

the car stops suddenly.

2
R R
CHAPTER

CHAPTER
M Massive oil tankers carry M

(a) Empty trolley (b) Full trolley crude oil ranging from 150 to
4 4
300 thousands tons. With full
Figure 2.21
power on, a tanker with a
very large inertia takes a long
2 Similarly, it is easier to stop an empty trolley than a full one if both are time to accelerate to its
moving at the same speed towards the shopper. maximum speed. The tanker
•Inertia = ‘Laziness’ literally, such that an object will not change its state of motion unless forced to do so. 3 The more mass an object has, the harder it is to change its state of might need to travel a
motion. distance of 10 km before
For example, coming to a stop even with
(a) It is more difficult to start moving a bucket filled with sand. the engine set into reverse
I am in (b) When both the buckets are swinging and an attempt is made to stop condition (propeller in
I am at rest, motion. them at the lowest point of the swing, it is more difficult to stop the reverse rotation).
I will always I will always It is not an easy job to turn
move at
bucket filled with sand.
stay at rest. the tanker around since the
constant
speed in a inertia of the crude oil is
ceiling
straight line. enormous. Supertanker
rope of officers need special training
the same in the handling of heavily
length It is harder
•Force is needed to change its velocity, i.e., to start it
loaded ships.
sand moving from
(a) to get the object to start moving, or
rest
(b) to make the object move faster, slower or change the direction of motion.

It is harder
to stop it
Hard-boiled or raw? empty bucket filled here with
bucket with sand your hand
On a table are two eggs, one raw and one hard-boiled.
Figure 2.22
How can the two eggs be distinguished?

(c) This shows that the bucket with more mass offers a greater resistance
Solution
to change from its state of rest or from its state of motion.
The answer is to spin the egg. 4 By the same reasoning, it is harder to start a bowling ball moving and
Spin each egg on its side. It is much easier to spin the hard-boiled egg. harder to stop it than a hollow rubber ball of the same size.
A hard-boiled egg is solid whereas a raw yolk floats in the liquid white
5 Thus, an object with a larger mass has a larger inertia.
in a raw egg. When a hard-boiled egg is spun, the solid contents turn
Mass is a measure of the inertia of a body.
together with the shell. When a raw egg is spun, the yolk and the liquid
white tend to remain at rest because of inertia. The reluctance to spin 6 However, inertia is a phenomenon. It has no unit even though it is
along with the shell makes the raw egg harder to spin. closely related to mass.

Forces and Motion 60 61 Forces and Motion


Effects of inertia
2.1
SPM SPM
’04/P3(A) ’06/P3(B)
Relationship between inertia and mass Animals such as dogs and If the bottom of the handle of a hammer is
Situation Procedure cats shake their bodies knocked downwards against a hard surface,
vigorously to dry their the loose head of the hammer tightens in its
Mrs Tan and her son Siao Yang are sitting on two 1 One end of the hacksaw blade is clamped by a
wet fur. The droplets of wooden handle. The hammer head continues
similar swings. G-clamp to a leg of a table as shown in Figure 2.24. with its downward motion after the handle
2 A 50 g plasticine ball is fixed to the free end of water on the fur tend to
continue in motion when has come to a stop. This causes the upper
the blade.
the fur are reversed in part of the wooden handle to slot deeper into
3 The free end of the blade is displaced horizontally the hammer head.
and released so that it oscillates. The time for 20 direction during shaking.
F complete oscillations, t20, is measured using a As a result, water droplets F
O stopwatch. This step is then repeated. The average are separated from the fur O
2

2
R of t20 is calculated. Then, the period of oscillation, and fall off. R
CHAPTER

CHAPTER
M M
Figure 2.23 t
T is determined by using T = 20 .
4 Mr Tan gives each of them a push while they are at 20 4
rest and later tries to stop them in the same position. 4 Steps 2 and 3 are repeated with plasticine balls of
Mr Tan finds that it is harder to push as well as to masses 75 g, 100 g, 125 g and 150 g.
stop Mrs Tan’s motion. 5 A graph of T against m is drawn.
Inference
Tabulation of data
The inertia of an object depends on its mass. The chilli sauce in a bottle
Hypothesis Table 2.3 can be forced out easily by
Applications
An object with a larger mass has a larger inertia Mass of Time of 20 oscillations, Period of When chased by a bull, run in a zigzag pattern. of the turning the bottle upside
(represented by a longer period of oscillation). load, t20 (s) oscillation, The larger inertia of the bull makes it more concept of down and giving it a quick
Aim t difficult for the bull to turn around inertia downward shake, followed by
m (g) t1 t2 Average T = 20 (s)
20 continuously. a sudden stop. The sauce
To investigate the relationship between mass and
continues its downward
inertia using an inertia balance 50 movement due to its inertia
Variables when the bottle is stopped
75
(a) Manipulated : mass of plasticine, m 100 causing the sauce to be
(b) Responding : period, T forced out of the
(c) Fixed : the stiffness of a hacksaw blade 125
Sumo wrestlers are heavy. bottle.
and the distance of the centre of 150 This is an advantage because the When the branch of an
the plasticine from the clamp larger inertia forms a greater apple tree is shaken, the
Graph
Notes resistance to his opponent who is apples fall to the ground.
T(s) trying to topple him. The apples which are
A larger inertia means it is more difficult for the
stationary tend to remain at
load to start or stop, thus making the time for one
rest when the branch is shaken.
oscillation, T longer. Therefore, the period of
As a result, the stalks are
oscillation represents the inertia.
strained and the apples break away
Apparatus/Materials Droplets of water on a
from the branch.
Hacksaw blade, G-clamp, stopwatch, and plasticine wet umbrella can be
balls of mass 50 g, 75 g, 100 g, 125 g and 150 g. m (g) spun off if the umbrella
Arrangement of apparatus is rotated vigorously
Figure 2.25
and stopped it
Experiment 2.1

Conclusion abruptly. Owing to


The graph of T against m in Figure 2.25 shows that inertia, water droplets
G-clamp
hacksaw blade the period increases with the mass of the load i.e. an on the umbrella
object with a larger mass has a larger inertia. continue to move even
plasticine
The hypothesis is valid. though the umbrella has
Figure 2.24 stopped spinning.

Forces and Motion 62 63 Forces and Motion


Ways of Reducing the Negative Effect of Inertia SPM
’07/P2
3 SPM
Clone

–1
’10
1 If a car crashes while travelling at a fixed velocity of 30 m s , for
example, the inertia of the passengers causes them to continue moving The diagram shows two trolleys P and Q with two R S
at 30 m s–1 until a force acts to change this speed. This is a identical wooden blocks R and S placed on them.
A Pushed to the right Pushed to the left
dangerous situation. Upon impact, the passengers will crash into Trolley P moves and collides with the stationary
the parts of the car immediately in front of them and suffer injuries trolley Q. B Pushed to the right Pushed to the right
(Figure 2.26). C Pushed to the left Pushed to the right
R S
2 Ways to reduce the negative effect of inertia in a car: D Pushed to the left Pushed to the left

(a) Safety belt P Q


Comments
Seat belts secure the driver The collision of the trolleys causes trolley P to slow
F and passengers to their seats. Direction of movement of P Stationary down while trolley Q to start moving. Due to F
O When the car stops suddenly, inertia, R continues to move fast to the right, and O
2

2
R R
the seat belt provides the Key : Right appeared to be pushed to the right while S, reluctant
CHAPTER

CHAPTER
M Left M
external force that prevents Figure 2.26 to move along with trolley Q, appeared to be
4 the driver or passengers from pushed to the left. 4
What happens to wooden block R and S?
being thrown forwards. Answer A
(b) Airbag system
Airbags mounted on the 2.3
dashboard or steering wheel
inflate automatically when a 1 In a bus moving with a uniform speed in a straight 4 A circus strongman slams a hammer and breaks a
collision occurs. This prevents line, a boy drops a steel sphere from rest outside the brick over the hand of a clown. The clown feels no
the driver or passengers from window. He observes that the steel sphere drops pain. Why?
crashing into the dashboard. vertically downwards. Explain. (Caution: Please do not attempt this at home.)
2 A durian is originally placed in a lorry as shown in
Figure (a). When the lorry starts to move with forward
acceleration, the durian rolls backwards. Later, when
3 Lorries that carry heavy loads utilise the following features to reduce the negative effects of inertia: the lorry stops suddenly, the durian rolls forwards as
shown in Figure (b). Explain the observations.
hammer
(a) Strong structure behind the driver’s cabin If a loaded lorry stops abruptly, its heavy load, for
strong iron structure
example, timber logs, will continue to move
forward towards the driver’s cabin because of its
massive inertia. A strong iron structure between
brick
the driver’s cabin and the load ensures the driver’s hand
on floor
safety.
(a)

(b) Subdivision of the mass to reduce its inertia The figure shows a liquid product being carried in 5 Suria runs along a track from P to R through Q while
holding a pail full of water. At which points of the
three separate compartments instead of one. This
track will more water likely to be spilt? Give your
reduces the effect of inertia of the liquid on the explanation.
walls of each container if the lorry stops suddenly.

(b)
Q
(c) Fastening of the objects to the carrier Big items such as furniture or large electrical 3 The figure below shows two blocks of the same
dimensions but of different materials. One is heavier R
appliances (the objects) must be securely fastened
than the other. Without lifting the two blocks, explain
to the lorry (the carrier) so that the objects become
one way to identify the heavier block.
part of the carrier. This will ensure that the objects
and the carrier always move and stop together.
The objects will not fall off the lorry when the lorry P
starts moving, or move forwards when the lorry
stops suddenly.

Forces and Motion 64 65 Forces and Motion


2.4 Analysing Momentum

2.4 Analysing Momentum Linear Momentum

1 Activity 2.2 also shows that it is always harder to stop a massive object Momentum and inertia are
What is Momentum? not the same.
moving at a high velocity.
1 If a loaded lorry and a car are moving at v 2 The above activity serves to explain a concept in physics called
Momentum Inertia
the same speed, it is more difficult for momentum.
the lorry to stop (Figure 2.27). 3 The linear momentum, p, of a m Inertia depends solely on
2 This is because the lorry possesses a mass, m, moving at a velocity, mass while momentum as
physical quantity, momentum, more v
v, is defined as the product of v
‘mass in motion’ depends on
than the car. mass and velocity. both mass and velocity.
3 All moving objects possess momentum. Figure 2.27
Figure 2.29
F 4 Activity 2.2 will help you to gain the idea F
O of momentum by comparing the effects Momentum = Mass × Velocity O
Unit = kilogram × metre per second
2

2
R of stopping two objects. ∴ p = mv = kg × m s–1
R
CHAPTER

CHAPTER
M M
4 The unit of momentum is kg m s–1.
4 5 Momentum is a vector quantity with the same direction as velocity. 4
Activ To compare the effects of stopping two objects in motion
ity 2.2 6 If the direction to the right is denoted as positive, an object moving to
the right possesses a positive momentum while an object moving to
Apparatus/Materials One steel ball and one wooden ball of the same diameter, 2 slabs of plasticine. the left will have a negative momentum.
Arrangement of apparatus steel

Note 15
steel steel
Note
Although both balls A ball of mass 0.8 kg strikes a wall at a velocity of (a) Take the direction to the right as positive.
100 cm wood
The steel ball have the same 10 m s–1 and rebounds at 6 m s–1. Momentum of the ball before striking the wall, p1
released from a diameter, the steel What is its momentum = mv
greater height strikes 50 cm 50 cm ball has a greater (a) before it strikes the wall, and = 0.8 × 10
the plasticine at a plasticine mass than the (b) after the rebound? = 8 kg m s–1
greater velocity. wooden ball.
Solution 10 m s–1 (b) Momentum of the ball after rebound, p2
(a) (b)
Figure 2.28 = mv
= 0.8 × (– 6)
(a) Two objects of the same mass moving at (b) Two objects of different masses moving at = – 4.8 kg m s–1
different velocities the same velocity Since direction to the
right is positive, direction
Procedure Procedure 6 m s–1 to the left is negative.
1 A steel ball is first released from a height of 1 A steel ball and a wooden ball of the same
50 cm and then from 100 cm above a slab of diameter are released from a height of 50 cm
plasticine as shown in Figure 2.28(a). above a slab of plasticine as shown in Figure Conservation of Momentum 4 To be precise, the Principle of conservation of
2 The depths and sizes of the cavities caused 2.28(b). momentum is true for a closed system.
by the steel ball on the slab are observed and 2 The depths and sizes of the cavities formed A closed system is one where the sum of
1 The term conservation is used if the total
compared. are observed and compared. external forces acting on the system is zero.
amount of matter or quantity remains the same
5 The principle shall be discussed in two
before and after the occurence of an event.
Observations Observations situations as shown in Table 2.4.
2 The Principle of conservation of momentum
The depth and size of the cavity caused by the The depth and size of the cavity formed by the states that: Table 2.4
steel ball released from a greater height is deeper steel ball is deeper and larger.
and larger. A collision An explosion
The total momentum of a system The total momentum of the The sum of the
Activity 2.2

Conclusion is constant, if no external force objects before a collision momentums


The moving balls produce an effect on the plasticine which is there to stop the motion. The greater the mass acts on the system. equals that after the remains as
or the velocity of the moving object is, the greater is the effect (the depth and size of the cavity), the greater is collision. zero after an
the momentum. 3 An example of an external force is friction. explosion.

Forces and Motion 66 67 Forces and Motion


SPM SPM
Collisions ’06/P1 ’08/P1
Activ To verify the principle of conservation of momentum in
ity 2.3 (a) elastic collisions, and (b) inelastic collisions
There are two types of collisions.
Apparatus/Materials
Collisions Ticker-timer, 12 V a.c. power supply, runway, 4 trolleys, wooden block, ticker tape, cellophane tape,
and plasticine.

Elastic collisions Inelastic collisions (A) Elastic collision


Arrangement of apparatus
Two objects collide and move apart after a Two objects combine and stop, or move together
spring-loaded piston
collision. with a common velocity after a collision. ticker-timer trolley A
ticker tape trolley B
mA
F m1 m2 m1 m2
mB
F
friction-compensated
m1 m2 m1 m2
O runway
O
2

2
R R
u1 u2 v1 v2 u1 u2 v
CHAPTER

CHAPTER
M M
• Momentum is conserved. • Momentum is conserved.
12 V a.c. wooden
4 • Total energy is conserved. • Total energy is conserved. power supply block 4
• Kinetic energy is conserved. • Kinetic energy is not conserved:
The total kinetic energy after the collision is less Figure 2.30
than the total kinetic energy before the collision. Procedure
Formula: Formula: 1 The apparatus is set up as shown in Figure 2.30.
m1 u1 + m2 u2 = m1 v1 + m2 v2 m1 u1 + m2 u2 = (m1 + m2) v 2 The runway is adjusted so that it is friction-compensated.
3 Two trolleys of equal mass are used. Trolley A with a spring-loaded piston is placed at the higher end of the
runway while trolley B is placed halfway down the runway.
4 A ticker tape is attached to trolley A and another to trolley B. Both ticker tapes are allowed to pass through the
ticker-timer.
5 The ticker-timer is switched on and trolley A is given a slight push so that it moves down the runway at
a uniform velocity and collides with trolley B.
6 After the collision, the two trolleys move separately.
Most collisions are inelastic as a significant amount of kinetic energy is converted to other forms of energy
during the collision. However, the collisions involving two objects that bounce off one another with little 7 From the ticker tapes, the velocities of trolleys A and B before and after the collision are calculated.
deformation during collision are approximate elastic collisions. Collisions between air molecules are elastic. 8 Assuming that the mass of each trolley is 1 unit, the momentum before and after the collision is calculated
Examples of approximate elastic collisions are as follows: and recorded in a table.
9 The experiment is repeated using:
(a) 1 trolley to collide with 2 stationary stacked trolleys,
Approximate elastic collisions (b) 2 stacked trolleys to collide with 1 stationary trolley.
Results
Collision of steel balls of equal mass Collision of two snooker balls of equal mass 1 Ticker tapes obtained:
Before collision After collision Before collision After collision (a) Trolley A
after during before
collision collision collision
stationary stationary
direction
of motion
x2 x1
P T x2 x1
QR S T P QRS vA = uA =
0.2 0.2

• Ball P is pulled to the side and then released so as to • Ball P is hit and moves with velocity u. It is observed (b) Trolley B
after during before
fall back and strike ball Q. It is observed that ball P that ball P stops when it collides with ball Q which collision collision collision
stops, but ball T swings out to the same height from moves away with velocity u.

Activity 2.3
which the ball P was released. • This shows that ball Q, after the collision, has the direction
• This shows that ball T possesses the same amount of of motion
same amount of momentum and kinetic energy as x3 uB = 0
momentum and kinetic energy as P before it struck ball P before the collision.
x3
ball Q. vB =
0.2

Figure 2.31

Forces and Motion 68 69 Forces and Motion


Tabulation of data Results
Ticker tape obtained:
Table 2.5 during
collision
before
after
Before collision After collision collision
collision

Initial total Final total direction


of motion
mA mB uA momentum, vA vB momentum,
x2 x1
mA uA mA vA + mB vB x2 x1
v = u =
0. 2 0.2
1 1
1 2 Figure 2.33
Tabulation of data
2 1 Table 2.6
F F
O O
Before collision After collision
2

2
R R
Initial Initial Initial total Final Final Final total
CHAPTER

CHAPTER
M Disscussion M
mass, velocity, momentum, mass, velocity, momentum,
4 1 The spring-loaded piston acts as a ‘springy buffer’ in the collision in order to make the trolley bounce off the mA u (mA u) (mA + mB) v (mA + mB) v 4
other one.
2 Strictly speaking, this collision is not a perfect elastic collision as part of the kinetic energy of the colliding 1 1+1=2
trolley changes to sound or heat energy during the collision. 1 1+2=3
2 2+1=3
Conclusion
From Table 2.5, it is found that: Conclusion
From Table 2.6, it is found that:
Total momentum before collision = Total momentum after collision
Total momentum before collision = Total momentum after collision
The principle of conservation of momentum is verified. The principle of conservation of momentum is verified.

(B) Inelastic collision 16


Arrangement of apparatus
An astronaut of mass 90 kg moves at a velocity of 6 m s–1 and bumps into a stationary astronaut
ticker-timer of mass 100 kg. How fast do the two astronauts move together after collision?
ticker plasticine
tape 90 kg 100 kg v
trolley A 100 kg
trolley B 6 m s–1 at rest 90 kg
12 V friction-compensated
a.c. power runway
supply

wooden block

Figure 2.32 (a) Before collision (b) After collision


Figure 2.34
Procedure
Solution
1 Some plasticine is pasted onto trolleys A and B (both without a spring-loaded piston) as shown in Figure 2.32.
This is an inelastic collision.
2 A ticker tape is attached to trolley A only.
Total momentum before collision = Total momentum after collision
3 The ticker-timer is switched on. Trolley A is given a gentle push so that it moves down the runway to collide
m1 u1 + m2 u2 = (m1 + m2)v
with trolley B which is stationary halfway down the runway.
(90 × 6) + (100 × 0) = (90 + 100)v
4 After the collision, trolley A attaches itself to trolley B and they move together.
540
Activity 2.3

Activity 2.3
5 From the ticker tape obtained, the velocity of trolley A before the collision, and the common velocity of v is the common velocity
v = of the two astronauts.
trolleys A and B after the collision are determined. The initial velocity of trolley B is zero. 190
6 The experiment is repeated using: = 2.8 m s–1
(a) 1 trolley to collide with 2 stationary stacked trolleys, –1
The two astronauts move at the speed of 2.8 m s after the collision.
(b) 2 stacked trolleys to collide with 1 stationary trolley.

Forces and Motion 70 71 Forces and Motion


SPM
Momentum and Explosions ’04/P2
17
A 50 kg skater is moving due east at a speed of 3 m s–1 before colliding into another skater of mass 1 Rifle
60 kg moving in the opposite direction at a speed of 7 m s–1. After the collision, the two skaters hold 2
The explosion creates a backward momentum on
on to each other. In which direction will they move? What is the speed of the two skaters? the rifle. This causes the rifle to recoil backwards.
3ms
–1
7ms
–1 Total momentum of the rifle
and the bullet is zero as they ar d
are stationary. ckw tum
v=? ba men v1
mo riflev 2 forward momentum
on on bullet
m1
m2

F 1 F
When the rifle is fired, the explosion of
O O
the gunpowder forces the bullet out of
2

2
R the barrel. A momentum in the forward R
(a) Before collision (b) After collision
CHAPTER

CHAPTER
M direction is created. M
Figure 2.35
4 4
Solution (a) Before explosion (b) After explosion
Total momentum before collision = Total momentum after collision Figure 2.36
m1 u1 + m2 u2 = (m1 + m2)v
50 × 3 + 60 × (–7) = (50 + 60) × v 2 Air escapes from a deflating balloon
150 – 420 = 110v upward
v = –2.5 m s–1 v is the common velocity
momentum
of the two skaters. m2
Momentum to the east taken as positive, so balloon 2
v2
momentum to the west is negative. Total momentum The balloon shoots upwards,
of the balloon is moving with an upward momentum.
–1 stationary
The two skaters will move to the west at a speed of 2.5 m s . (u = 0) zero as it is
stationary. 1
v1 Air has mass and moves with a velocity.
This creates a momentum in the
m1 downward direction.
downward
18 momentum

A trolley of mass 3 kg moving at a velocity of 2 m s–1 collides with another trolley of mass 0.5 kg (a) Before explosion (b) After explosion
which is moving at a velocity of 1 m s–1 in the same direction. If the 0.5 kg trolley moves at a Figure 2.37
velocity of 2.5 m s–1 in the same direction after the collision, what is the velocity of the 3 kg trolley?
3 An explosion is a closed system which does not involve any external force—that is, the total
Solution momentum is conserved in an explosion.
2 m s–1 1 m s–1 v=? 2.5 m s–1 ∴ Total momentum before explosion = Total momentum after explosion
3 kg 0.5 kg 3 kg 0.5 kg 0 = m1v1 + m2v2
Rearranging the formula:
m1v1 = –m2v2
(a) Before collision (b) After collision where v1 and v2 are of opposite directions.
The collision is elastic. If we ignore the direction,
Total momentum before collision = Total momentum after collision m1v1 = m2v2
m1 u1 + m2 u2 = m1 v1 + m2 v2
3 × 2 + 0.5 × 1 = 3 × v + 0.5 × 2.5 as the two moment have the same magnitude.
6.5 = 3v + 1.25
3v = 6.5 – 1.25
5.25
v=
3 • Momentum to the left = Momentum to the right
Bear in mind that the two velocities
= 1.75 m s–1 • Momentum upwards = Momentum downwards are in opposite directions.
–1
The 3 kg trolley moves at a velocity of 1.75 m s in its original direction. • Momentum forwards = Momentum backwards

Forces and Motion 72 73 Forces and Motion


Activ To verify the principle of conservation of momentum in an 19
ity 2.4 explosion
Jane and John go ice skating. With their skates on,
Apparatus/Materials Jane and John push against each other on level ice.
4 trolleys, 2 wooden blocks, a hammer, and a metre rule. Jane, of mass 50 kg, moves away at a velocity of
Notes 3 m s–1 to the right. What is John’s velocity if he is
75 kg?
The positions of the wooden blocks are adjusted so that each trolley collides with the corresponding
wooden blocks at the same time, t. From the equation d = vt, the magnitude of the velocity v is Solution
directly proportional to the distance d, i.e., v d (if t is constant). Thus, the distance d travelled by the This is a closed system since the external force, i.e.,
trolley represents the velocity of the trolley. friction, is negligible.
Arrangement of apparatus Procedure Let the velocity of John be v.
F F
Total momentum Total momentum
O (a) Before explosion 1 The apparatus is arranged as shown in Figure =
after explosion before explosion Figure 2.39 O
2

2
R 2.38 (a). R
release pin
m1 v1 + m2 v2 = 0
CHAPTER

CHAPTER
M wooden M
trolley B trolley A block 2 Two trolleys A and B of equal mass are placed in 50 × 3 + 75v = 0
4 contact with each other on a smooth surface. The 4
–150
mB mA spring-loaded piston in trolley B is compressed. v = _____ The minus sign indicates that
3 The release pin on trolley B is given a light tap to 75 John moves to the left,
–1
= –2 m s opposite to Jane’s motion.
dB dA release the spring-loaded piston which then
pushes the trolleys apart. The trolleys collide with Alternative Method
(b) After explosion the wooden blocks.
Momentum to the right = Momentum to the left
spring- 4 The experiment is repeated and the positions
vB
loaded
vA m1 v1 = m2 v2
piston of the wooden blocks are adjusted so that both Just ignore the sign as the two
50 × 3 = 75 × v
trolleys collide with them at the same time. skaters are moving in opposite
v= 2 m s–1 directions.
5 The distances dA and dB are measured and
recorded.
Figure 2.38 6 The experiment is repeated using ∴ John moves to the left at a velocity of 2 m s–1.
(a) 1 trolley with 2 stacked trolleys,
(b) 3 stacked trolleys with 1 trolley.

Tabulation of data 20
The results of the experiment are recorded in Table 2.7.
Figure 2.40 shows trolley A, with a weight attached, placed in contact with trolley B on a smooth surface.
Table 2.7
weight attached to trolley A
Before explosion After explosion release pin
Initial total Mass of Mass of Velocity of Velocity of Final total
momentum trolley A, trolley B, trolley A, trolley B, momentum, A B

mA mB dA (–dB) mA dA + mB (– dB)
d1 = 1 d2 d2
3
0 1 1 0
Figure 2.40
0 1 2 0
0 3 1 0 When the release pin of trolley B is tapped lightly, the Solution
spring-loaded piston pushes the two trolleys to move Applying m1 d1 = m2 d2:
Discussion
in opposite directions. The two trolleys touch the 1 m1 is the total
1 Total momentum before explosion = 0 (because both trolleys are stationary) wooden blocks simultaneously. Trolley A moves one- m1 × d2 = m × d2 mass of trolley A
Total momentum after explosion = mAdA + mB (–dB) 3 and the weight.
as d represents v, third the distance moved by trolley B. Given that the
Activity 2.4

2 Table 2.7 shows that mA dA + mB (–dB) = 0 and v is a vector.


two trolleys are of equal mass, m kg, find the mass of m1 = 3m
∴ Total momentum after explosion = Total momentum before explosion the weight, in terms of m, that is attached to trolley A. The mass of the weight, m′
Conclusion = 3m – m
Momentum is conserved in an explosion. = 2m

Forces and Motion 74 75 Forces and Motion


2 Jet engine
21 1 2 3 4
Air from the atmosphere is drawn into In the combustion chamber, The hot gases formed expand rapidly
Harfeez alights a boat at a vm s–1
the engine and compressed by a and are forced out of the nozzle at
kerosene fuel burns vigorously
velocity of v m s–1 and lands compressor before it is forced into the with the compressed air. high speed through the turbine which
on the dock. v' = ? combustion chamber at high pressure. rotates the compressor.
The boat bounces backwards
with a speed of v′. 3 combustion 4 turbine blade
2 compressor
If the masses of Harfeez and chamber
the boat are m and 3m
respectively, find the speed
of the boat (ignoring friction Figure 2.41
F due to water). 5 F
O 1
jet of
O
2

2
R Solution air exhaust R
CHAPTER

CHAPTER
M intake gases M
m1 v1 + m2 v2 = 0
4 (m × v) + (3m × v′) = 0 4
3mv′ = –mv
1 nozzle
v′ = – v
3
6 moves forwards fuel injector
1 –1
The speed of the boat = v m s
3
6
In accordance with the principle of the conservation of 5
momentum, a forward momentum for the engine is produced. The ejected high-speed exhaust gases create
Application of the Conservation of Momentum The plane thus flies forwards. a backward momentum.

1 Rocket Figure 2.43

1 A rocket carries liquid 5


Demonstrations to Show the Principle of Propulsion in Jet Engines and Rockets
hydrogen and liquid
oxygen. 5 In accordance with the
The following demonstrates the principle in rockets and jet engines.
principle of conservation of
momentum, the rocket gains
Jet engine Rocket
a forward momentum and
2 The mixture of hydrogen 1 2 1 5 upward
moves forwards at high thread cylindrical
fuel and oxygen burns liquid velocity. weight
1
3 plastic
vigorously in the hydrogen air bottle
front
(as fuel)
combustion chamber. water
1 3 plastic tube
liquid bicycle rubber stopper 2
oxygen pump
elastic back tripod stand
downward
3 The gases formed band
4
expand rapidly and are
forced to discharge 1 A weight is placed on the trolley. 1 One-third of a plastic bottle is filled of water.
2
through the exhaust combustion 4 A backward momentum 2 As the thread is cut, the weight is 2 The bottle is then plugged tightly with a rubber stopper with a
chamber
nozzle at a high is created. thrown to the back with a plastic tube.
velocity. momentum. 3 Air is pumped into the bottle.
3 3 This in turn creates a forward 4 When the pressure of the air in the bottle is sufficient, it will
hot exhaust
gases at momentum which causes the push the stopper out together with the water. A downward
high velocity trolley to move forwards. momentum is produced.
4
5 This in turn creates an upward momentum which moves the
plastic bottle upwards.
Figure 2.42

Forces and Motion 76 77 Forces and Motion


2.4
• The principle of the conservation of momentum occurs in nature — the squid uses it to propel itself in the water. 1 A pigeon of mass 120 g is flying at a velocity of 5 A butterfly rests on a leaf floating on the surface of a
The squid moves forward by discharging a jet of water from its body. An equal and opposite momentum created thus 2 m s–1. What is its momentum? pond.
propels the squid in the opposite direction.
5 cm s–1
2 A bull of mass 250 kg is moving at a momentum of
750 kg m s–1. Find its velocity.

3 During training, Othman fires a pistol of 1.5 kg mass. 3 cm s–1

The butterfly then starts moving to the tip of the leaf


F • The shower of burning fragments from an exploding fireworks launched into the sky is governed by the principle of the at a speed of 5 cm s–1 relative to the water. The leaf, F
O conservation of momentum. The symmetrical pattern indicates that the total momentum is conserved. stationary in accordance with the principle of the conservation O
2

2
R of momentum, moves at 3 cm s–1 relative to the R
CHAPTER

CHAPTER
M water in the opposite direction. If the mass of the leaf M
is 8 g, determine the mass of the butterfly.
4 (a) Before shooting 4
6 Boat A and boat B are moving at a speed of 2 m s–1
and 1 m s–1 respectively before the two collide head
on. The masses of boats A and B (including the
passengers) are 150 kg and 250 kg respectively.
30 g
• The large volume of water that rushes out from a water hose with a very high speed has a large momentum. In
accordance with the principle of the conservation of momentum, an equal and opposite momentum is created causing
1.5 kg
the fireman to fall backwards. Thus, several firemen are needed to hold the water hose.
(b) After shooting

A bullet with a mass of 30 g is released at a (a) Before collision


velocity of 300 m s–1. What is the recoil velocity of
the pistol? 0.5 m s–1 v=?

4 Hizam and his son Jamal are at an ice rink.

A B

(b) After collision


4 SPM
Clone
If boat A bounces back with a velocity of 0.5 m s–1,
’11
what is the velocity of boat B?
The diagram shows three identical theme park coins P Q R 7 Sau Fei and Siew Ling, each with a mass of 60 kg and
on a straight line, with Q and R touching each other. 49.5 kg respectively, are standing at rest on an ice
A Moves Stationary Stationary rink. Sau Fei throws a ball of mass 0.5 kg towards Siew
P is given a velocity and collides Q.
B Stationary Stationary Moves Ling.
Q R C Moves Moves Stationary
token
P token
D Moves Stationary Moves
token

Comments
The momentum of P is totally transferred to R
What will be observed? Jamal with a mass of 20 kg is moving at a velocity of
through Q.
2 m s–1 while Hizam with a mass of 60 kg, is directly
Therefore, P stops and R moves with the velocity of
behind Jamal and moving at 6 m s–1. Hizam decides
P before collision while Q remains stationary. to pick Jamal up and continues moving without
Answer B stopping. Determine the final velocity of Hizam and What is the recoil velocity of Sau Fei if the velocity of
Jamal. the ball is 8 m s–1? What is the velocity of Siew Ling
after she receives the ball?

Forces and Motion 78 79 Forces and Motion


2.5 Understanding the Effects of a Force

2.5 Understanding the Effects of a Force


The effects of force are applicable in the games below.
What is Force? (a) Football
1
3 Ben kicks the football to Sani from a corner
F during a football match. He needs to apply
F A force is a push Hazuri manages to catch the ball.
or a pull. He needs to apply a force to stop the ball. a force to make the stationary ball move.
Also, as his boot is in contact with the ball,
Pressing a switch the force applied deforms the ball.

F
Lifting objects Hazuri
F Ben F
F F
O O
2

2
R F R
CHAPTER

CHAPTER
M M
COKE
Pulling off the ring of a soft drink tin Stretching a chest expander Kicking a football Sani
4 4

1 When you push or pull on an object, you need to know


(a) the strength or magnitude of your force, and
(b) the direction in which you are pushing or pulling.
2 Therefore, force is a vector quantity since it has both magnitude and
2

10
0

9
direction. spring balance
Sani's header produces a force on the ball to change its direction.
3 A spring balance which is used to measure the weight (or the
gravitational pull on the object) can be used to measure the magnitude
of a force. For example, the force pulling a wooden block can be (b) Baseball
measured as shown in Figure 2.44. Figure 2.44

The Effects of a Force

The shape of a tube of toothpaste A spring lengthens or compresses A plastic ruler can be bent when
changes when you press on it. when you stretch or compress it. a force is exerted on it.

1 A force can change the shape of an object (deformation of an object). 3 2 1


If the ball is not hit, it is The force from the bat deforms the ball A pitcher applies a force when

stopped by the catcher. and reverses the direction of the ball. he throws the baseball.
Effects of force

2 A force can change the original state of motion (either at rest or in motion) of an object.
(a) To move a stationary object (b) To stop a moving object (c) To change the direction of
motion of an object Relationship between Acceleration, Mass and Force

1 Consider two identical speedboats A and B at thrust is applied, speedboat P accelerates


rest on the sea. Different engine thrusts are slower than Q.
applied to accelerate the boats. However, 3 Thus, the three quantities: acceleration, mass
speedboat A which applies more engine thrust and force, are related.
increases the velocity at higher rate. 4 The relationship between the three quantities
A pushing force is required to Friction will stop the bicycle A tennis player hits a fast-moving
2 Consider two identical speedboats, P and Q can be determined in the following experiments.
move a stalled car. when the boy stops pedalling. tennis ball to return it to his opponent.
with P heavily loaded. When the same engine

Forces and Motion 80 81 Forces and Motion


Results
2.2
Ticker tape chart:
Velocity (cm per tentick) Velocity (cm per tentick) Velocity (cm per tentick)
Relationship between acceleration and force applied on a constant mass
elastic cord attached to the trolley and stretched to a the gradient
Situation is tripled
fixed length represents one unit of force acting on the the gradient
Figure 2.45(a) shows car A and car B of the same is doubled
trolley.
mass at the same starting line. Car B is a sports car.
The engine capacity of sports car B is much bigger Apparatus/Materials
than car A. (A car with a bigger engine capacity can
provide greater engine thrust.) Trolley, 3 identical elastic cords, runway, ticker-
Figure 2.45(b) shows that sports car B has built up a timer, carbonised ticker tape, cellophane tape, 12 V (a) One elastic cord (b) Two elastic cords (c) Three elastic cords
higher velocity than car A after 3 seconds. a.c. power supply and a wooden block. (a) (1 unit of force) (b) (2 units of force) (c) (3 units of force)
F F
O Arrangement of apparatus Figure 2.47 O
2

2
R B (greater engine capacity) R
1 The gradient of the line passing through the dot at the top of each successive strip increases with the number
CHAPTER

CHAPTER
M M
ticker-timer stretched elastic cord (1 unit of force)
ticker tape
of cords.
4 A 2 This indicates an increase in acceleration when the force is increased. 4
friction-compensated
a.c. power runway Tabulation of data Graph
supply
trolley
Table 2.8 1 The graph of a against F is a straight line passing
wooden block through the origin as shown in Figure 2.48.
(a) t = 0 Units of force Acceleration, This shows that: a F (if m is constant).
(a)
applied, F a (m s–2) acceleration, a
B
greater
two stretched elastic cords (2 units of force)
1 a1
engine
thrust [same extension as (a)] 2 a2
A (force)
3 a3
force, F
O
(b) t = 3 s Figure 2.48
Conclusion
Figure 2.45
The acceleration of an object is directly proportional to the force applied if the mass is constant.
(b)
Can you make an inference about this situation? The hypothesis is valid.
Figure 2.46
Inference
The acceleration of an object depends on the force Procedure
which acts on it. 1 A friction-compensated inclined runway is
2.3
Hypothesis prepared.
2 The apparatus is then set up as shown in Figure Relationship between acceleration and the mass of an object under a constant force
When the mass of an object is constant, the greater 2.46(a). Situation
the force applied, the greater its acceleration. 3 The ticker-timer is switched on and the trolley is
Figure 2.49(a) shows two similar lorries, A and B in front of a traffic light. When the light turns green, both drivers
Aim pulled down the runway by an elastic cord
step on the accelerator simultaneously with the same pressure to provide the same engine thrust, F.
attached to the hind post of the trolley.
To investigate the relationship between acceleration Figure 2.49(b) shows that within 3 seconds, the empty lorry has built up a higher velocity than the heavy one.
4 The elastic cord is stretched until the other end
and the force applied on a constant mass.
is level with the front end of the trolley. The
Variables length is maintained as the trolley runs down the

Experiments 2.2 & 2.3


(a) Manipulated : force applied, F runway. B
B
(b) Responding : acceleration of trolley, a 5 The ticker tape obtained is cut into strips of
Experiment 2.2

full-loaded F
(c) Fixed : mass of trolley, m, and the 10-tick. A tape chart is constructed and the lorry
stretching force of an elastic cord acceleration, a, is determined. A
A
6 The experiment is repeated with 2, and 3 elastic empty
lorry F
Notes cords to double and triple the pulling force to the
The force in this experiment is the stretching force in same constant extension as when one elastic cord
an elastic cord used to pull the trolley. A length of is stretched. (a) t = 0 (b) t = 3 s
Figure 2.49

Forces and Motion 82 83 Forces and Motion


Inference 5 The elastic cord is stretched until the other end
Graph 1
is level with the front end of the trolley. The The graph of a against shows a straight line
The acceleration of an object depends on its mass. m
length is maintained as the trolley runs down the
acceleration, a
Hypothesis runway. passing through the origin as shown in Figure 2.52.
6 The ticker tape obtained is cut into strips of 1
When the force applied on an object is constant, the This shows that: a (if F is constant).
10-tick. A tape chart is constructed and the m
greater the mass of the object, the smaller its
acceleration. acceleration, a, is determined.
7 The experiment is repeated using 2 trolleys (with Conclusion
Aim
a second trolley stacked on the first trolley) and The acceleration of an object is inversely
To investigate the relationship between acceleration 3 trolleys. The elastic cord is stretched to the O proportional to its mass when the force acting
and the mass of an object under a constant force. same fixed length as in the first experiment. Figure 2.52 on it is constant.
Variables Results The hypothesis is valid.
F F
(a) Manipulated : mass of trolley, m Ticker tape chart:
O O
(b) Responding : acceleration of trolley, a
2

2
R Velocity (cm per tentick) R
(c) Fixed : force applied by an elastic cord, F
CHAPTER

CHAPTER
M M
Notes
4 4
(a) The mass in this experiment is represented by the
number of identical trolleys used.
(b) The constant force is applied by stretching the Newton's Second Law of Motion 22
elastic cord with the same extension for each (a) One trolley (1 unit of mass)
repetition of the experiment. 1 From Experiment 2.2: a F A force of 10 N acts on an object of mass 5 kg on a
Velocity (cm per tentick)
smooth floor. Find its acceleration.
Apparatus/Materials 1
From Experiment 2.3: a a=?
Ticker-timer, 12 V a.c. power supply, 3 trolleys, m
the gradient 10 N
is halved The two results are combined.
elastic cord, runway, wooden block, ticker tape and
cellophane tape. F
a
Arrangement of apparatus m
(b) Two trolleys (2 units of mass) k is a constant. Figure 2.54
ticker-timer stretched elastic cord or F ma
ticker tape Velocity (cm per tentick) ∴ F = kma Solution
a.c. power
friction-compensated runway 2 The unit of force is Newton, N. F = 10 N, m = 5 kg
supply
trolley
the gradient is 3 In order to make the formula as simple as possible, F = ma
one third of (a)
we make k = 1 by defining a force of 1 N as: 10 = 5a
wooden block
a = 2 m s–2
(a)
(c) Three trolleys (3 units of mass) 1 N is the force which gives a mass of 1 kg
stretched elastic cord
F [same extension as in (a)] an acceleration of 1 m s–2.
Figure 2.51

2 units of mass
The gradients of the lines joining the uppermost dots
for successive strips decrease as the number of 1 kg a = 1 m s-2

trolleys used increases. This indicates that 1N Fnet = ma is the mathematical expression of Newton's
acceleration decreases as mass increases. second law of motion.
(b) This law states:
Tabulation of data
Figure 2.50 Figure 2.53 The net force on an object is proportional to the rate of
Table 2.9 change of momentum.
Procedure
Mass of Inverse Change in momentum
1 A friction-compensated inclined runway is Acceleration, F = kma Force (net)
Time
trolley, m of mass,
prepared. 1 a 1 N = k × 1 kg × 1 m s–2 mv – mu The net force on an
Experiment 2.3

Experiment 2.3
(Number of Fnet object causes the
2 The apparatus is then set up as shown in Figure 2.50. m (m s–2) k=1 t
trolleys) object with a mass
3 A ticker tape is attached to the trolley and passed
1 1.00 a1 ∴ F = ma Fnet m
(v – u) of m to change its
through the ticker-timer. t velocity from u to v.
4 The ticker-timer is switched on and the trolley is 2 0.50 a2
pulled down the inclined runway by an elastic Newton's second Fnet ma v–u
3 0.33 a3 a=
law of motion Fnet = kma t
cord attached to the hind post of the trolley.

Forces and Motion 84 85 Forces and Motion


Balanced forces (Fnet = 0, a = 0)
23
–152
A car of mass 1200 kg travelling at 15 m s–1 comes a = 2 × 30 F1 F2
F net = 0
to rest over a distance of 30 m. Find Retardation is
(as no force
–2 deceleration.
(a) the average retardation, and a = – 3.75 m s acting on it)
(b) the average braking force. ∴ Retardation = 3.75 m s–2 F1 = F2
(b) F = ma From Fnet = ma:
Solution Negative sign indicates
= 1200 × (–3.75) 0 = ma
retardation force or
u = 15 m s–1, v = 0 m s–1, s = 30 m opposing force. ∴ a = 0 (since mass, m cannot be zero)
= – 4500 N
(a) Using the formula v2 = u2 + 2as:
0 = 152 + 2a(30) Average braking force = 4500 N

F
Object in motion F
O
Object at rest (v ≠ 0, and the object is moving O
2

SPM SPM SPM SPM


(v = 0 m s–1)

2
R Balanced Forces and Unbalanced Forces ’03/P1 ’05/P1 ’07/P2(C) ’08/P1 at constant velocity) R
CHAPTER

CHAPTER
M M
1 In general, there may be several forces acting on the object (whether a single force or several
4 4
on the mass, whether parallel or anti-parallel, forces are acting on it).
or in different directions. For example, For example,
2 Thus, the force, F, must be replaced with the Balanced Forces
net or resultant force when there are several
forces acting on the mass. 1 When the forces acting on an object are
5N
balanced, they cancel each other out (that is,
2 cm s–1
Some prefer to use ΣF = ma
net force = 0). PHYSICS
Fnet = ma where ΣF is the sum of the 2 The object then behaves as if there is no force 200 N Ffriction
force vectors. acting on it.
3 Since Fnet = 0, the acceleration of the object,
Ffriction
where a is in the direction of the net or a = 0. Thus, the object remains at rest or moves Linda pushes a book on a table with a force of 5 N.
resultant force. at constant velocity when there is no net force Zamhari pushes a heavy cupboard with a force of The book moves with a uniform velocity of 2 cm s–1.
3 However, for simplicity, F = ma is always used, acting on it. This is Newton’s first law of motion. 200 N, but the cupboard does not move. Find the frictional force acting on the book.
bearing in mind that F is the net force acting 4 Examples of balanced forces: Find the frictional force acting on the cupboard. Solution
Solution Using Fnet = ma: because the book
(a) Balanced forces on a stationary gymnast (b) Balanced forces on a car moving at a constant velocity Using Fnet = ma: because the But Fnet = 0 since a = 0 moves with a
cupboard does uniform velocity
R But Fnet = 0 since a = 0 5 – Ffriction = 0
not move
∴ 200 – Ffriction = 0 Ffriction = 5 N
Ffriction = 200 N (The frictional force here is known as dynamic
(The frictional force here is known as static friction) friction)
air resistance, G
thrust, T

Note: In both situations, the frictional forces are equal to the forces applied.
T = G + Fr

weight, W W=R
friction, Fr
W
SPM
Effect of Balanced Forces and Unbalanced Forces on an Object
W=R
’08/P2(A)
reaction force, R
from beam • There are 3 horizontal forces acting on a car moving
at a constant velocity. The forward thrust, T, Balanced forces (Fnet = 0, a = 0)
provided by the car engine is balanced by the
frictional force on the wheels and the air resistance. lift from wings, L 1 Balanced forces on an aircraft allow it to
• The weight of the gymnast, W, is balanced by the
reaction force, R, from the beam. T = G + Fr move at constant velocity at a constant
• The two forces are of equal magnitude but opposite altitude.
in direction. drag or air resistance, G
2 The engine thrust is balanced by the drag due to
• The weight of the car, W, is vertically balanced by engine thrust, T
• Without the beam (that is, no reaction force), the the reaction force, R, from the road. air resistance while the weight of the aircraft is
T = G
gymnast will fall to the ground because of her weight, W
W = L
balanced by a lift from the wings. The lift from
weight. W=R the wings is discussed in Chapter 3.

Forces and Motion 86 87 Forces and Motion


Unbalanced forces (Fnet ≠ 0, a ≠ 0)
26 5 SPM
Clone
’05
lift from wings, L
A shopper pushes a trolley with a force of 20 N. The Two forces F1 and F2 act on a wooden block which is
engine thrust, T trolley with a mass of 5 kg, moves at a uniform placed on a table. The friction between the table and
velocity of 1 m s–1. He then increases his force to the block is 3 N.
drag or air accelerate the trolley. What force should he apply in
weight, W resistance, G
order to give the trolley an acceleration of 2 m s–2? F2 F1

1 When the forces acting on an object do not cancel out each other, a net force known as unbalanced force is Solution
table
acting on the object.
2 Unbalanced forces produce an acceleration to the mass on which the forces are acting. 20 N Fʹ = ?
Which pair of forces F1 and F2 will accelerate the block?
3 However, the object will accelerate in the direction of the net force. 1 m s–1
F a = 2 m s–2 F
O 4 When an airplane is moving at a constant velocity, if the pilot increases the engine thrust, the forces acting F1 (N) F2 (N) O
2

2
R R
horizontally are no longer balanced. There is a net force forwards and the plane will accelerate in the A 4 7
CHAPTER

CHAPTER
M (a) (b) M
forward direction. B 8 5
4 For (a): Ffriction = 20 N 4
C 6 4
For (b): Fnet = ma
24 F′ – Ffriction = ma
The trolley is moving D 9 5
with uniform velocity,
Figure 2.55 shows 5000 N
Solution F′ – 20 = 5 × 2 i.e. the force applied
equals friction acting Comments
a small rocket of
upward F′ = 30 N on the trolley.
force from Since the upward force is greater than the downward
mass 300 kg at the engine
force by 2000 N, the rocket accelerates upwards. The block will move with acceleration if the forces
2000 N
point of take-off. Neglecting the mass of the exhaust gases, the upward acting on it are unbalanced.
Find its initial mass resultant
launching acceleration is given by: If the difference between the forces F1 and F2 is
acceleration. 300 kg upward
force Fnet = 5000 N – 3000 N
27 greater than 3 N, then there is an acceleration.
weight = 2000 N Mr Brown whose mass is 70 kg, performs as a human Answer D
Fnet = ma cannonball at a circus. He is propelled from a 1.6 m
F 2000
3000 N
a = mnet = long cannon. He is in the barrel of the cannon for 1.2 s.
300 6 SPM
Clone
Figure 2.55 = 6.7 m s–2 ’07

The figure below shows a car of mass 1200 kg


moving at an acceleration of 2 m s–2.
25
John pushes a 12 kg carton with a force of 50 N. • When a floor is smooth, frictional force is nil.
acceleration

velocity increasing
Figure 2.57
2 m s–2 F ILE
AG (a ≠ 0) Find the average net force exerted on him.
FR
F1 = 50 N
Ffriction = 0 smooth surface Solution If the frictional force acting on the car is 750 N, find
Fnet = 50 – Ffriction
First, we need to find the acceleration of Mr Brown its engine thrust.
• When an object on a rough floor moves at uniform in the barrel. Then, apply F = ma to find the net force A 750 N C 2400 N
velocity with a horizontal force acting on it, friction on him. B 1350 N D 3150 N
frictional force, Ffriction
is equal in magnitude to the applied force (but u = 0 m s–1, s = 1.6 m, t = 1.2 s Solution
Figure 2.56 acts in the opposite direction). 1
Using s = ut + at 2 : Use Fnet= ma since two forces are acting on the car.
If the carton moves with an acceleration of 2 m s , –2
2
Let T be the engine thrust of the car.
what is the frictional force acting on the carton? constant velocity 1
F (a = 0) 1.6 = 0 + a (1.2)2 T – Friction = ma
Solution ILE 2 T – 750 = 1200 × 2
AG
FR
Fnet = ma
rough surface
a = 1.6 × 2 = 2.22 m s–2 T = 2400 + 750
50 – Ffriction = 12 × 2 Ffriction 1.44 = 3150 N
F F F = Ffriction
Ffriction = 50 – 24 = 26 N Fnet = ma = 70 × 22.2 = 1554 N Answer D

Forces and Motion 88 89 Forces and Motion


2.6 Analysing Impulse and Impulsive Force
Newton’s Third Law of Motion
Newton’s third law of motion states that:
Impulse and Impulsive Force

If object A exerts a force, F on object B, then object B will exert an equal but opposite force, –F on
F
object A. In other words:
To every action there is an equal but opposite reaction. F

Everyday phenomena that are governed by Newton’s (b) When a man paddles with a backward force, +F
third law of motion: (action), the reaction force, –F, pushes the boat
(a) When a boy presses on the wall with a force, F, the forwards.
wall presses on his hands with a normal reaction • When a player kicks a football, his boot is in
F contact with the ball for a time t. F
force, –F.
O O
• During the time t, an average force F acts on the • When a tennis racket hits an oncoming tennis ball,
2

2
R R
reactions, –F ball which makes the ball fly off with a the tennis racket delivers a large force that acts on
CHAPTER

CHAPTER
M M
action,+F momentum. the ball for a short time t. The ball bounces off in
reactions,
4
–F action, +F
• Thus, the force F, acting for a period of time t, the opposite direction. Again, there is a change in 4
produces a change in momentum to the ball, momentum.
since the ball with a mass of m acquires velocity v • The change in momentum is due to the force F
(c) The principle used in rockets and jet engines can also after the time t. acting on the object for a time t.
be explained by Newton’s third Iaw of motion. The
Note: The action and reaction forces of Newton’s third action that pushes the exhaust gases out through the
law act on different objects whereas two nozzle results in a forward force (reaction force) that
balanced forces act on the same object. propels the rocket or jet engine forwards.
F = ma
F=m
( )
v–u
t
substitute a=
v–u
t

2.5 Ft = mv – mu mv – mu SPM
’07/P1
F = t
1 What force is needed so that an object with a mass of 6 (a) When a 2 kg block is pushed with a force of
3 kg has an acceleration of 2 m s –2? 12 N to the right, it accelerates with an
acceleration of 3 m s–2. In which direction does Impulse = Change in momentum
2 A wooden box of mass 2 kg is placed on a smooth Change in momentum
plane. If a force of F is applied to the box, it moves at friction act? Find its magnitude. Impulsive force =
Time taken
an acceleration of 3 m s–2.
a = 3 m s–2
Find the acceleration of another box with mass of unit = N s or unit = kg m s–1
6 kg if the same force is exerted on it. unit = N
12 N
3 Puan Zaitun pushes a trolley of mass m kg with a force 2 kg • Impulse is defined as the product of a
of 30 N. The trolley moves with a uniform velocity of force, F and the time interval, t during which
1 m s–1. When she doubles her force, the trolley • Impulsive force is defined as the rate of change
the force acts. of momentum during a collision or explosion.
accelerates at 2 m s–2. Find the mass of the trolley.
Inpulse = Force × Time
4 Faizal rides a bicycle at a constant speed of 16 m s–1. (b) An additional force of 20 N to the left then acts =F×t
He stops pedalling and the bicycle stops completely • Impulsive force is the large force produced in a
on the block. In which direction does friction • Impulse is a vector quantity and has the same
after 6 s. Given the total mass of Faizal and his now act? Find the new acceleration of the collision or explosion that happens in a short
direction as the force that causes the change in time of impact.
bicycle is 72 kg, find the average opposing force on block.
Faizal and his bicycle. momentum.
5 A race car of mass 1200 kg accelerates from rest to a
12 N 20 N
SPMvelocity of 72 km h–1 in 8 s.
Clone 2 kg
’07
(a) Find the acceleration of the car.
(b) Find the net force acting on the car. • Both formulae can be used to solve numerical problems.
• Both are vector quantities.

Forces and Motion 90 91 Forces and Motion


Rebound and Impulse
Effects of Time on Impulsive Force SPM
’09/P1
SPM
’09/P2/(A)

1 An object might rebound from a wall, or stick to it without rebounding after striking it.
2 In which situation will the wall exert a greater impulse? Look at Example 28. 1 From F = mv – mu 1
t F
t
28 = Change in momentum ⇒ t small, F large
Time of impact
⇒ t large, F small
A tennis ball and a piece of mud with the same mass Solution If the change in momentum is constant, then:
(0.060 kg) which are moving at 9 m s–1 strike a wall. Take the direction to the right as positive. From F 1
The mud sticks to the wall while the ball rebounds at t
6 m s–1. Find the impulse on each object. For the mud: For the tennis ball:
Impulse Impulse 2 A student throws a raw egg at a high speed at a wall, and another egg
F
9m s-1
= Change in momentum = Change in momentum against a towel held by his friends. In which case will the egg break? F
O mud = mv – mu = mv – mu O
2

2
9 m s-1 towel
R R
= 0 – 0.06 × (–9) = 0.06 × 6 – 0.06 × (–9)
CHAPTER

CHAPTER
M egg
egg M
tennis ball = 0 + 0.54 = 0.36 + 0.54
4 6 m s-1 = 0.54 N s = 0.90 N s 4

Figure 2.58 pile of towel

The egg is stopped by the wall in a very short time The egg thrown against a towel falls on a few
interval, resulting in a large force which causes it towels stacked below and does not break.
3 Thus, a greater impulse is exerted on an object if it rebounds after a collision. to shatter. The movement of the egg is stopped in a longer
time interval, resulting in a small force.
29
Figure 2.59(a) shows a 2 kg wooden block initially at If the force acts for 3 s, In both cases, the eggs are stopped. Thus, the change in momentum is the same.
rest on a smooth surface. A force of 8 N is applied on (a) what is the impulse on the block?
the wooden block. (b) what is the velocity of the wooden block after 3 s? 3 To understand the effect of time in a collision, look at the following example.
u=0 v Solution
8N 8N
2 kg 2 kg
(a) Impulse = F × t = 8 × 3 = 24 N s Save your knees!
t=0s t=3s (b) Impulse = Change in momentum
Joginder (mass, m = 50 kg) jumps down from a wall. He lands on a cement ground at a velocity of 6 m s–1.
Ft = mv – mu
(a) (b)
24 = 2v – 0
Figure 2.59 Joginder bends his knees upon landing. Joginder didn’t bend his knees upon landing.
v= 12 m s–1
The time taken to stop his motion is 1.0 s. The time taken to stop his motion is 0.05 s.

30
A player spikes an oncoming volleyball moving Solution u = 6 m s–1

towards him at a speed of 5 m s–1 to reverse its direction u = 6 m s-1


(a) Take the direction away from the hand as positive. v = 0 m s–1
at a speed of 20 m s–1. The mass of the ball is 0.36 kg. Impulse = Change in momentum v = 0 m s-1
= mv – mu Solution
5 m s -1 20 m s-1
= 0.36 × 20 – 0.36 × (–5) Take the downward direction as positive. Solution
= 0.36 × 20 + 0.36 × 5 m = 50 kg, u = 6 m s–1, v = 0 m s–1, t = 1.0 s Take the downward direction as positive.
F
= 9Ns mv – mu m = 50 kg, u = 6 m s–1, v = 0 m s–1, t = 0.05 s
F =
mv – mu t The negative sign indicates the mv – mu 50 × 0 – 50 × 6
(b) F = 50 × 0 – 50 × 6 F= =
t = force acts as an opposing force t 0.05
1 that reduces the momentum
9 of the object. = –6000 N
Figure 2.60 300 = = –300 N
t The impulsive force acting on Joginder’s leg is 6000 N!
(a) Find the impulse on the ball. 9 This force is great enough to break Joginder’s legs.
t= The impulsive force that acts on Joginder’s leg is 300 N.
(b) If the average force acting on the ball is 300 N, 300
how long is the time of contact between the hand
and the ball? = 0.03 s This example clearly shows that an effective way to reduce the impulsive force is to lengthen the collision time.

Forces and Motion 92 93 Forces and Motion


Increasing the Impulsive Force by Reducing the Time of Impact Reducing the Impulsive Force by Increasing the
Time of Impact In sports, the effects of
impulsive forces are reduced
In all the sports shown below, the time of impact with
It should be noted that to prevent injuries to
the ball is very small and the impulsive force
impulsive forces always athletes. Thick mattresses
produced is large.
exist in pairs. In the figure Polystyrene and cardboard egg with soft surfaces are used
on the right, the change in containers are stiff but in events such as the high
momentum of the tennis compressible. They will jump and pole-vaulting so
ball produces a large absorb and reduce impulsive that the time of impact
impulsive force on the force by lengthening the on landing is extended,
racket which reacts to give time of impact. thus reducing the
rise to an equal but resultant impulsive force.
F2 F1
FF opposite impulsive force to F F
polystyrene
OO the ball (this is in O O
2

A footballer kicking A golfer driving a

2
RR a football golfball with a club accordance with Newton’s R R
CHAPTER

CHAPTER
MM The use of padding in certain sports equipment M M
third law of motion).
Both the racket and the like baseball gloves, goalkeeping mitts, pole-
44 vaulting pits, boxing gloves, and gymnastic mats 4 4
tennis ball are deformed cardboard egg carton
temporarily due to the is to prevent injuries to players by reducing the
large force being exerted on impulsive force.
A batter hitting a A tennis player hitting a each other.
baseball tennis ball

A massive hammer head


moving at a fast speed is
brought to rest upon hitting
The effects of The effects of
t
t time ont the
the nail. The large change in (a) Baseball glove (b) Boxing glove
small small

F F
momentum within a short
time interval produces a time on the
large
large impulsive force which magnitudela of magnitude of
the impulsive F small
large large In baseball, a player must catch
drives the nail into the wood. the impulsive the ball in the direction of the
force force motion of the ball. If the ball is
m
mo
caught
c
cau by stopping it in its path,
t impulsive force acting on the
the
hand will be considerable.
h
ha
pile driver Moving
M
Mo his hand backwards
when
wh catching the ball
In prolongs
pr the time for the
pestle
pile construction,
tion momentum change to occur so
mo
the pile driver the impulsive force is reduced.
is raised to a
certain height Playgrounds are covered with a coarse
before it is fabric material which prolongs thee time
An expert in karate can When a boxer sees that
released. The of impact when the children fall,l, thus
split a thick wooden slab mortar his opponent’s fist is
momentum reducing the impulsive force.
with his bare hand which going to hit his head, he
acquired by Food such as chillies and
is moving at a very fast will move his head
the massive onions can be pounded using a
speed. The momentary backwards or duck.
pile driver is greatest just before it mortar and pestle (both made
contact between the This will increase the
hits the pile. The subsequent from stone). The pestle is
fast-moving hand and stopping time, hence
change in momentum that brought downwards at a fast
the wooden slab reducing the average
occurs in a short time interval speed and stopped by the
produces a large force on his head since
results in a large impulsive mortar in a very short time.
impulsive force which the momentum change
force which drives the pile into This produces a large coarse fabric
splits the wooden slab. material will be longer.
the ground. impulsive force which crushes
the food.

Forces and Motion 94


94 95 Forces and Motion
31 Comments landing to lengthen the time of impact; reducing the
The impulsive force is inversely proportional to the impulsive force.
A tennis player hits an oncoming 0.060 kg tennis ball Students are always tempted to find the acceleration, a
time of impact. The athlete bends his legs upon
with a velocity of 60 m s–1. The ball bounces off in the and then use F = ma to find the impulsive force.
mv – mu Answer B
opposite direction at 90 m s–1. By using F = , you save the step of finding a,
t
90 m s-1
which is not necessary. Try using the latter formula if the
125 N question mentions the impulsive force, and
acceleration is not stated. 2.6
60 m s-1
1 A force F acts on a 6 kg object at rest on a smooth 6 A baseball of mass 0.14 kg moving at 40 m s–1 is
surface. If the velocity of the object increases to struck by a bat and rebounds at 60 m s–1.
F
7 SPM
Clone
2 m s–1 in 6 s, what is the value of F ? F
’07
O 2 An object of mass 2 kg is acted on by a force which O
2

2
R Figure 2.61 The front and rear sections of a car are designed to R
causes the velocity of the object to increase from 40 m s-1 60 m s-1
CHAPTER

CHAPTER
M crumple easily. What is the reason? M
1 m s–1 to 9 m s–1. What is the impulse on the object?
Find the time of impact between the racket and the A To increase the impact time
4 ball if the impulsive force acting on the ball is 125 N. 3 The figure shows a helicopter dropping a box of mass 4
B To increase the impulse
60 kg which touches a sandy ground at a velocity of
Solution C To decrease the momentum 12 m s–1. The box takes 2 seconds to stop after it
D To increase the frictional force (a) (b)
touches the ground.
Take the direction to the right as positive. Given that the time of contact is 5 × 10–2 s, find the
F = 125 N, m = 0.060 kg, v = 90 m s–1, u = –60 m s–1 Comments force exerted on the ball.
mv – mu Change in momentum
F= Impulsive force =
t Time 7 In a crash test, a car of mass 1500 kg crashes into a
0.06 × 90 – 0.06 × (–60) When the time of impact is prolonged, the impulsive wall at 15 m s–1. It rebounds at 2 m s–1.
125 =
t force is reduced.
5.4 + 3.6
t= Answer A 15.0 m s–1
125
= 7.2 × 10–2 s

(a) Before collision


8 SPM
Clone

32 ’09

An athlete bends his legs upon landing in a long 2 m s-1


Tiger Woods hits a golf ball of mass 0.045 kg at a
velocity of 30 m s–1. jump event as shown in the Figure 2.63.
What is the magnitude of the impulsive force on the
box?
m = 0.045 kg (b) After collision
4 Beckham kicks a ball with a force of 1500 N. The
time of contact of his boot with the ball is 0.008 s. If the collision time is 0.18 s, find the force exerted
30 m s–1 What is the impulse delivered to the ball? If the mass on the car.
of the ball is 0.5 kg, what is the velocity of the ball?
5 A 0.045 kg golf ball strikes a wall at a speed of 8 A pole-vaulter (mass, m = 50 kg) falls onto a foam
Figure 2.62 30 m s–1 and rebounds at a speed of 20 m s–1. mattress which exerts a force of 250 N on him over a
If the time of impact is 0.005 s, what is the average time interval of 2.0 s.
30 m s-1
impulsive force applied on the ball by the club?
Solution Figure 2.63
u=?
m = 0.045 kg, u = 0, v = 30 m s–1, t = 5 × 10–3 s foam
The purpose of bending his legs is to reduce the 20 m s-1 250 N
mv – mu mattress
Impulsive force, F = A impulse on his feet
t
B impulsive force on his legs What is the impulse on the ball? If the force on the
(0.045 × 30) – (0.045 × 0)
= C velocity before landing ball is 500 N, find the contact time of the ball with
5 × 10–3 D time of impact between his legs and the sand. the wall.
= 270 N Find his velocity just before landing on the mattress.

Forces and Motion 96 97 Forces and Motion


2.7 Being Aware of the Need for Safety Features in Vehicles

lmportance of Safety Features in Vehicles SPM


’07/P1

1 It is important to increase the time interval Safety seat belts


of collision to reduce the impulsive force in • Prevents the passenger from being
thrown forward or out of the car. Slows
an accident. This concept was explained in down the forward movement of the
Section 2.6. passenger when the car stops abruptly.
2 Figure below explains the safety features in
the design of vehicles. Headrest
• To push the driver's head so that it moves
F together with the rest of his body when a
O stationary car is knocked from behind and
2

R Shatter-proof windscreen pushed forward suddenly. This will prevent


• Prevents the windscreen
CHAPTER

M a severe whiplash injury to the neck.


from shattering and reduce
injuries of passengers caused
4 by the shards of glass.
.
Automatic airbag
• Acts as a cushion
for the head and
body in an
accident and thus
prevents injuries to
the driver and front
passenger.

Tyres with tread


• Drains away water in
its grooves when the
car is on a slippery
road. The car has
more contact on
the road.

Strong steel struts


• Prevents the collapse of the front and back doors
of the car into the passenger compartment.
Bumper Also gives good protection from a side-on collision.
• Absorbs the impact
in minor accidents,
thus preventing
damage to the car. Anti-lock braking system (ABS)
• Prevents the wheels from locking when the
brakes are applied suddenly.
• Allows a car to stop quickly in slippery road
conditions and prevents the car from skidding.

Padded dashboard
• Increases the time interval of collision,
thereby reducing the impulsive force
Crumple zone
produced during impact.
• Increases the time interval of impact so that the resultant impulsive
force is reduced. Therefore, only little energy is transferred to the
inside of the car, and the passengers sustain minimal bodily injuries.
• The car is less likely to rebound upon impact. This reduces the
momentum change or impulse.

Forces and Motion 96


98

You might also like